SlideShare a Scribd company logo
1 of 92
Download to read offline
2 4
Contemporary Communication Systems 1st Edition Mesiya Solutions Manual
Download:http://testbanklive.com/download/contemporary-communication-systems-
1st-edition-mesiya-solutions-manual/
Chapter 2
2.1 Consider the signals di splayed in Fi gure P2.1.Show that each of these signals can be
expressed as the sum of r ectangular Π (t ) and/or triangular Λ (t ) pulses.
Figure P2.1
Solution:
a. x (t )
⎛ t ⎞ ⎛ t ⎞
1 = Π ⎜ ⎟ + Π ⎜ ⎟
⎝ ⎠ ⎝ ⎠
b. x (t ) = 2 Λ
⎛ t −3 ⎞
− Λ
⎛ t −3 ⎞
2 ⎜
6
⎟ ⎜
2
⎟
⎝ ⎠ ⎝ ⎠
c. x (t ) = ... + Π
⎛
t +
11 ⎞
+ Π
⎛
t +
7 ⎞
+ Π
⎛
t +
3 ⎞
+ Π
⎛
t −
1 ⎞
+ Π
⎛
t −
5 ⎞
+ Π
⎛
t −
9
⎞
+ ...3 ⎜
2
⎟ ⎜
2
⎟ ⎜
2
⎟ ⎜
2
⎟ ⎜
2
⎟ ⎜
2
⎟
⎝ ⎠ ⎝ ⎠ ⎝ ⎠ ⎝ ⎠ ⎝ ⎠ ⎝ ⎠
∞
= ∑ Π ⎡⎣ t − ( 2 n + 0.5 )
n =−∞
1
2
2
2
2
2
x (t ) = 2 ⎢ 2
+
2
s gn (
t ) ⎥
2
d. x (t ) = ... + Λ
⎛ t +4 ⎞
+ Λ
⎛ t ⎞
+ Λ
⎛ t −4 ⎞
+ ...4 ⎜
2
⎟ ⎜
2
⎟ ⎜
2
⎟
⎝ ⎠ ⎝ ⎠ ⎝ ⎠
∞
⎛ t −4 n ⎞
= ∑ Λ ⎜ ⎟
n =−∞ ⎝ 2 ⎠
2.2 For the signal x (t ) in Figure P2.1 (b) plot the following signals
a. x2 (t − 3)
b. x2 ( −t )
c. x (2t )
d. x2 (3 − 2t )
Solution:
x (t − 3)
1
x2 ( − t )
1
0
1 2 3 4 5 6 7 8 9 −6 − 5 − 4 − 3 −
2 −1
0 t
1 2 3
x (2t )
1
x (3 − 2t )
1
0 t
1 2 3 4 5 6 −
2−1
0 t
1 2 3 4
2.3 Plot the following signals
a. x1 (t ) = 2Π (t / 2) cos ( 6π t )
⎡ 1 1 ⎤b.
⎣ ⎦
2
t
)
(t
4x
t
tt
) t( 1
x
)t)
(t(4
4x
x
tt
4
2
3
1
T
T
1
T
x1(t)
x4(
t)
"
2
c. x3 (t ) = x2 ( −t + 2)
d. x (t ) = s i n c ( 2t ) Π ( t / 2 )
Solution:
2 x (t )
1. 5
21
0. 5
0
-0. 5
0 t
1 2 3 4 5 6
-1
-1. 5
-2
1
0. 8
-1 -0. 8 -0. 6 -0. 4 -0. 2 0 0. 2 0. 4 0. 6 0. 8 1
t 0. 6
x (t )
2
"
0 t
1 2 3 4
0. 4
0. 2
0
-0. 2
-0. 4
-1 -0. 8 -0. 6 -0. 4 -0. 2 0 0. 2 0. 4 0. 6 0. 8 1
t
2.4 Determine whether the following signals are periodic. For periodic signals,
determine the funda me ntal period.
a. x1 (t ) = si n(π t ) + 5 cos( 4π t / 5)
Solution:
sin(π t ) is periodic with period T =
2π
= 2 . cos( 4π t /
5)
π
is periodic with period
T
2π 5 1
2 =
4π / 5
=
2
. x (t ) is peri odic if the ratio can be written as ratio of
2
integers. In the present case,
T 2 × 2 41
= =
5 5
3
2
1
o 1 2 3
T
3
2
2
Therefore, x (t ) is periodic with fundame ntal period T such that
1 o
T = 5T = 4T = 10
o 1 2
b. x (t ) = e j 3 t
+ e j 9 t
+ cos (12t )
Solution:
e j 3 t
is periodic with period
T
2π j 9 t
=
3
. Similarly, e is periodic with period
T =
2π
. cos(12t ) is periodic with period T =
2π
=
π
. x (t ) is periodic with
2
9
3
2π
12 6
2
funda me ntal
period
T = L C M ( T , T , T ) =
3
.
c. x3 (t ) = s i n( 2π t ) + cos(10t )
Solution:
sin( 2π t ) is periodic with period T =
2π
1
2π
= 1 . cos(10t
)
is periodic with period
T =
2π
=
π
. x (t ) is peri odic if the ratio
T1
can be written as ratio of integers.
2
10 5
3
In the present case,
T 1 × 5 51
= =
T2
π π
Since π is an irrational number, the ratio is not rational. Therefore,
periodic.
d. x (t ) = cos
⎛
2π t −
π ⎞
+ s i n(5π t )
x (t ) is not
4 ⎜
4
⎟
⎝ ⎠
Solution:
cos
⎛
2π t −
π ⎞
is periodic with period
T
2π
= = 1 . sin(5π t ) is periodic with
⎜
4
⎟ 1
2π⎝ ⎠
period T 2π
4 T
=
5π
2
=
5
. x (t )
is
peri
odic
if
the
ratio
T1
can be written as ratio
of
2
integers. In the present case,
4
T2
T 1 × 5 51
= =
2 2
Therefore, x (t ) is periodic with funda me ntal period T such that
4 o
T = 2T = 5T = 2
o 1 2
2.5 Classify the following signals as odd or even or neither.
a. x (t ) = − 4t
Solution:
x ( − t ) = 4t = − ( −4t ) = − x
(t ) . So
x (t ) is odd.
b. x (t ) = e
− t
Solution:
e
− t
= e
− − t
.
So
x (t ) is even.
c. x (t ) = 5 cos(3t )
Solution:
Since cos(t ) is even, 5 cos(3t ) is also even.
d. x (t ) = s i n(3t −
π
)
2
Solution:
x (t ) = si n(3t −
π
) = − cos(3t ) which is even.
2
e. x (t ) = u (t )
Solution:
u (t ) is neither even nor odd; For example, u (1) = 1 but u ( −1) = 0 ≠ − u (1) .
5
1
1
2
2
T
2
f. x (t ) = si n( 2t ) + cos( 2t )
Solution:
x (t ) is neither even nor odd; For example,
x (π / 8)
=
2 but x ( −π / 8) = 0 ≠ − x (π / 8) .
2.6 Determine whether the following signals are energy or power, or neither and
calculate the corresponding ener gy or power in the signal:
a. x1 (t ) = u (t )
Solution:
The normalized average power of a signal x (t ) is defined as
1
P = li m
T /2
∫ u (t )
2
d t = li m
1
T / 2
1 T 1
∫ 1 d t = li m =
x1 T →∞ T T →∞ T− T /2 0
T →∞ T 2 2
Therefore, x (t ) is a power signal.
b. x (t ) = 4 co s ( 2π t ) + 3 co s ( 4π t )
Solution:
4 cos ( 2π
t )
is a power signal. 3 co s ( 4π t ) is also a power signal.
Since
x (t ) is
sum of two power signals, it is a power signal.
1
Px = li m2 T →∞
T /2
∫
− T /2
4 cos ( 2π t ) + 3 cos (
4π t )
2
dt = lim
1
T →∞ T
T /2
∫
− T /2
4 cos( 2π t ) + 3 cos ( 4π t ) dt
1
= li m
T /2
⎡16 cos 2
( 2π t ) + 9 cos 2
( 4π t ) + 24 cos( 2π t ) cos ( 4π t ) ⎤d t
T →∞ T ∫ ⎣ ⎦
− T /2
Now
li m
1
T /2
∫
8
16
co
s 2
( 2π t )dt = li m
T /2
∫
8
T
[1
+
co
s(
4
π
t )
]dt
=
li
m
=
8T →∞ T
li m
1
T →∞ T
− T /2
T /2
∫
− T /2
T →∞ T
9 cos 2
( 4π t )dt = 4.5
− T
/2
T → ∞ T
6
T
⎦
x
⎛ − ⎞
⎝ ⎠
⎛ − ⎞
⎝ ⎠
3
4
2
4
24
T /2
24
T /2
li m
T →∞ T ∫− T /2
cos( 2π t ) cos ( 4π t )dt =
lim
T →∞
∫− T /2
⎡⎣ co s ( 6π t ) + co s ( 2π t ) ⎤dt = 0
Therefore,
P = 8 + 4.5 = 12.52
1
c. x (t ) =3
t
Solution:
T /2
E = li m 1 / t
2
dt = li m
T /2
⎛ 1
t −2
dt = li m −
T /2
⎞ 2
= li m = 0
x3 T → ∞ ∫− T /2
T /2
T →∞ ∫− T /2
T / 2
T →∞ ⎜
⎝ t ⎟− T /2 ⎠
T →∞
⎜
T / 2
⎟
T /2
1
P = li m 1 / t
2
dt = li m1
1 ⎛ 1
t −2
dt = li m −
⎞ 1 2
= li m = 0
x3 T →∞ T ∫− T /2
T →∞ T ∫− T /
2
T → ∞ T ⎜
⎝ t ⎟− T /2 ⎠
T → ∞ T
⎜
T / 2
⎟
x (t ) is neither an energy nor a power signal.
d. x (t ) = e − α t
u (t )
Solution:
E = li m
T /2
e − α t
u (t ) dt = li
m
T /2 ⎛ e −2α t T /2
e −2α t
dt = li m −
⎞ 1
= li m − e − α T
+ 1
1
=
x
T → ∞ ∫
T →∞
∫
⎜
T →∞ 2α
⎟
2α T → ∞
( ) 2α
4
⎜ ⎟
− T /2 0 ⎝ 0 ⎠
Thus x (t ) is an energy signal.
e. x5 (t ) = Π (t / 3) + Π (t )
Solution:
5
x (t )
2
1
− 3/ 2 3 / 2 t
7
5
6
2 2 t j 10 t
∫
4 t
T /2
6
7 ∑ ⎣
T /2
−1/2 1/ 2 −1/ 2
E x
= li m5 T →∞ ∫
− T /2
Π (t / 3) + Π (t )
2
dt
=
∫ 1d t
+
−3 / 2
∫
−1/2
4 d t
+
∫ 1d t
− 3 /2
= 1 + 4 + 1 = 6
Thus x (t ) is an energy signal.
f. x (t ) = 5e ( −2 t + j 10 π t )
u (t )
Solution:
E x
= li m6 T →∞
T /2
∫ 5e ( −2 t + j 10 π t )
u (t ) dt =
li m
T → ∞
T /2
∫
T / 2
2
5e −
e π
) dt = li m 2 5 e −
dt
T →∞
− T /2 0 0
⎛
= 25 li m ⎜
−
e −4 t ⎞
⎟ =
25
li m ( − e −2 T
+ 1) =
25
T →∞ ⎜
⎝ 4 ⎟ 40 ⎠
T →∞ 4
Thus x (t ) is an energy signal.
∞
g. x (t ) = Λ ⎡ ( t − 4 n ) / 2
n
=−∞
Solution:
x (t ) is a periodic signal with period T = 4 .7 o
To /2 1 1
1
P = ⎡ Λ
22
t / 2 ⎤ dt = 1 −
t
2
dt =
1
1 − 2t + t 2
dt
x7
T ∫ ⎣ ( ) ⎦ T ∫ ( )
2 ∫ ( )
o − To /2
1 ⎛
1
t 3
⎞
o 0 0
1 1 1
= ⎜ t − t 2
+ ⎟ =
⎛
1 − 1 +
⎞
=
2 3 2
⎜
3
⎟
6
⎝ ⎠ 0
⎝ ⎠
2.7 Evaluate the following expressions by us ing the properties of the delta function:
a. x1 (t ) = δ (4t ) s i n(2t )
Solution:
1
δ (4t ) =
4
δ (t )
1 1
x (t ) = δ (t ) s i n ( 2t ) = δ (t ) s i n ( 0) = 01
4 4
8
4
4
b. x (t ) = δ (t ) cos
⎛
30π t +
π ⎞
2 ⎜
4
⎟
⎝ ⎠
Solution:
x (t ) = δ (t ) cos
⎛
30π t +
π ⎞
= δ (t ) cos
⎛
0 +
π ⎞
= cos
⎛ π ⎞
δ (t )2 ⎜
4
⎟ ⎜
4
⎟ ⎜
4
⎟
⎝ ⎠ ⎝ ⎠ ⎝ ⎠
c. x3 (t ) = δ (t )s i nc (t + 1)
Solution:
x3 (t ) = δ (t )s i nc (t + 1) = δ ( t )s i nc ( 0 + 1) = δ ( t )s i nc (1) = δ ( t ) × 0 = 0
d. x (t ) = δ (t − 2) e − t
s i n ( 2.5π t )
Solution:
x (t ) = δ (t − 2 ) e − t
s i n ( 2 . 5π t )
= δ (t − 2) e −2
si n ( 2.5π × 2) = δ (t − 2) e −2
sin (π ) = 0
e. x5 (t ) =
∞
∫ δ ( 2t )s i nc (t ) d t
−∞
Solution:
x (t ) δ ( 2t )si n c (t
) d t
1
δ (t )si n c (t ) d t
∞ ∞
5 = ∫ =
2 ∫
−∞ −∞
1
δ (t )sinc ( 0) d t
1 1
δ (t ) d
t
∞ ∞=
2 ∫ =
2 ∫ =
2
−∞ −∞
f. x6 (t ) =
∞
∫ δ (t − 3) cos (t ) d t
−∞
Solution:
x6 (t ) =
∞ ∞
∫ δ (t − 3) cos (t ) dt = cos (3) ∫
−∞ − ∞
δ (t − 3) dt = cos (3)
9
1 ⎛ 4 ⎞ 3 t
− ×⎛ ⎞
τ
g. x (t )
∞
δ (2 t )
1
dt
7 = ∫−∞
−
1 − t 3
Solution:
∞
1
∞
1
x (t ) = ∫ δ ( 2 − t ) d t = ∫ δ (t − 2 ) d t7
1 t 3
1 t 3
−∞
− −∞
−
δ (t 2)
1
d t
1
δ (t 2) d t
1
∞ ∞= ∫ −
1 − 23
= −
7∫
− = −
7
−∞ −∞
h. x8 (t ) =
∞
∫ δ (3t − 4) e −3 t
d t
−∞
Solution:
∞ ∞
x (t ) = ∫ δ (3t − 4 ) e −3 t
dt = ∫ δ ⎜ t − ⎟ e −
dt8
3 ⎝ 3 ⎠−∞ −∞
1
∞
= δ t −
4 ⎞
e
3
4
3
dt =
e −4 ∞
δ
⎛
t −
4
dt =
e −4
∫ ⎜ ⎟ ∫ ⎜ ⎟
3 ⎝ 3 ⎠ 3 ⎝ 3 ⎠ 3
−∞ −∞
i. x9 (t ) = δ '(t ) ⊗ Π (t )
Solution:
∞
x (t ) (t τ )δ '(τ )dτ ( 1)
d
(t τ )
9 = ∫ Π − = −
d
Π −
−∞ τ = 0
= Π '
(t ) = δ ( t + 0.5 ) − δ ( t − 0.5 )
2.8 For each of the following continuous-time systems, determine whether or not the system is (1)
linear, (2) time-invariant, (3) me moryless, and (4) casual.
a. y (t ) = x (t − 1)
Solution:
The system is linear, time-invariant, causal, and has me mory. The system has
me mory because current value of the output depends onthepreviousvalueof
theinput. The system is causal because current value of the output does not
depend on future inputs. To prove linearity, let
response of the system to x (t ) is
x (t ) = α x1 ( t ) + β x2 (t ) . The
10
1 1 2 2
y (t ) = x (t − 1) = α x1 (t − 1) + β x2 ( t − 1)
= α y1 (t ) + β y 2 (t )
where x (t ) ⎯T
⎯→ y (t ) and x (t ) ⎯T
⎯→ y (t ) .
To show that the syst em is time-invari ant,
let input. The corresponding output is
x1 (t ) = x (t − t o ) be the system
y1 (t ) = x1 (t − 1) = x (t −
1 − t
o ) = y (t − t o )
b. y (t ) = 3 x (t ) − 2
Solution:
The system is nonlinear, time-invariant, causal, and me moryless. The system is
me moryless because current value of the output depends only on the current
value of the input. The system is causal because current value of the output does
not depend on future inputs.
To prove nonlinearity, let
to x (t ) is
x (t ) = α x1 ( t ) + β x2 (t ) . The response of the system
y (t ) = 3 x (t ) − 2 = 3 [α x1
(t ) + β x2
(t ) ] − 2
≠ α y1 (t ) + β y 2 (t ) = α [ 3 x1 (t ) − 2 ] + β [ 3 x2 ( t ) − 2 ]
To show that the syst em is time-invari ant,
let input. The corresponding output is
x1 (t ) = x (t − t o ) be the system
y1 (t ) = 3 x1 (t ) − 2 = 3 x
(t − t
o ) − 2 = y (t − t o )
c. y (t ) = x (t )
Solution:
The system is nonlinear, time-invariant, causal, and me moryless. The system is
me moryless because current value of the output depends only on the current
value of the input. The system is causal because current value of the output does
not depend on future inputs.
To prove nonlinearity, let
x (t ) is
x (t ) = α x1 ( t ) + β x2 (t ) . The response of the system to
11
1 1 o o
y (t ) = α x1 (t ) + β x2 (t ) ≤ α x1 (t ) + β x2 (t )
Because
α x (t ) + β x (t ) =
α
x (t ) + β x (t ) ≠ α y (t ) + β y (t ) fo r all α an d β ,
1 2 1 2 1 2
the system is nonlinear.
To show that the syst em is time-invari ant,
let input. The corresponding output is
x1 (t ) = x (t − t o ) be the system
y (t ) = x (t ) = x (t −
t
) = y (t − t )
1 1 o o
d. y (t ) = [ co s ( 2t ) ] x (t )
Solution:
The system is linear, time-variant, causal, and memoryless. The system is me
moryless because current value of the output depends only on the current value
of the input. The system is causal because current value of the output does not
depend on future inputs.
To prove nonlinearity, let
to x (t ) is
x (t ) = α x1 ( t ) + β x2 (t ) . The response of the system
y (t ) = [ cos ( 2t ) ] x (t ) = [ cos ( 2t ) ] [α x1
(t ) + β x2
(t ) ]
= α [ cos ( 2t ) ] x1 (t ) + β [ cos ( 2t ) ] x2 ( t ) = α y1 ( t ) + β y 2 (t )
To prove time-variance, let
corresponding output is
x1 (t ) = x (t − t o ) be the system input. The
y (t ) = [ cos ( 2t ) ] x (t ) = [ cos ( 2t ) ] x ( t − t ) ≠ y (
t − t ) = cos
2 ( t − t
o )
x ( t − t o )
e. y (t ) = e x ( t )
Solution:
The system is nonlinear, time-invariant, causal, and me moryless. The system is
me moryless because current value of the output de pends only on the current
value of the input. The system is causal because current value of the output does
not depend on future inputs.
12
1 2
1
1 1 o o o o
o
To prove nonlinearity, let
to x (t ) is
x (t ) = α x1 ( t ) + β x2 (t ) . The response of the system
y (t ) = eα x1 ( t ) + β x2 ( t )
= eα x1 ( t )
e β x2 ( t )
≠ α y (t ) + β y (t ) = α e x1 ( t )
+ β e x2 ( t )
for all α and β
To show that the syst em is time-invari ant,
let input. The correspondi ng output is
x1 (t ) = x (t − t o ) be the system
y (t ) = e x1 ( t )
= e x ( t − to )
= y (t − t )
f. y (t ) = t x (t )
Solution:
The system is linear, time-variant, causal, and me moryless. The system is me
moryless because current value of the output de pends only on the current value
of the input. The system is causal because current value of the output does
not depend on future inputs. To prove nonlinearity, let x (t ) = α x1 ( t ) + β x2 (t ) .
The response of the system to x (t ) is
y (t ) = t [α x1 (t ) + β x2 ( t ) ] = α t x1 ( t ) + β t x2 (t ) = α y1 (t ) + β y 2 ( t )
To prove time-variance, let
corresponding output is
x1 (t ) = x (t − t o ) be the system input. The
y (t ) = t x (t ) = t x ( t − t ) ≠ y (t − t ) = ( t − t ) x ( t − t )
t
g. y (t ) = ∫ x ( 2τ )
dτ
−∞
Solution:
The system is linear, time-invariant, causal, and has me mory. The system has
me mory because current value of the output depends only on the past values
of the input. The system is causal because current value of the output does not
depend on future inputs. To prove linearity, let
response of the system to x (t ) is
x (t ) = α x1 ( t ) + β x2 (t ) . The
13
1 ∫ 1 ∫ o
t t t
y (t ) = ∫ [α x1 ( 2τ ) + β x2 ( 2τ ) ] dτ = α ∫ x1 ( 2τ ) dτ + β ∫ x2 ( 2τ ) dτ = α y1 (t )
+ β y 2 (t )
−∞ −∞ −∞
To check for time-variance, let
corresponding output is
x1 (t ) = x (t − t o ) be the system input. The
t t
y (t ) = x ( 2τ ) dτ = x [ 2(τ − t ) ] dτ
=
t − t
o
∫
x ( 2α ) d α = y ( t − t o )
−∞ −∞ −∞
2.9 Calculate the output y (t ) of the LTI system for the followi ng cases:
a. x (t ) = e −2 t
u (t ) and h (t ) = u (t − 2) − u (t − 4)
Solution:
h (t ) = u (t − 2) − u (t − 4) = Π
⎛ t −3 ⎞
⎜
2
⎟
y (t ) h (τ ) x
(t
τ )
dτ
⎝ ⎠
e 2( t −τ )
u (t τ )
τ−3
dτ
∞ ∞
= − = − − Π
⎛ ⎞
∫ ∫ ⎜
2
⎟
−∞ −∞
For t < 2 , there is no overlap and
⎝ ⎠
y (t ) = 0.
t t − 2
e−2τ
t −
2
1 e −2 ( t − 2 )
For 2 ≤ t < 4
,
y (t )
=
∫ e −2( t −τ )
dτ
= ∫
e −2τ
dτ = =
−
−2 2
2 0 0
For t ≥ 4 ,
4 4
4
e 2τ
e 8
− e4 ( e 4
− 1)
y (t ) = ∫ e −2( t −τ )
dτ = e −2 t
∫ e 2τ
dτ
=e −2 t
= e −2 t
= e −2 ( t − 2 )
2 2 22 2 2
⎧1 − e −2( t − 2)
⎪
, 2 ≤ t ≤ 4
⎪ 2
⎪ ( e 4
− 1)y (t ) = ⎨ e −2( t − 2)
,
⎪ 2
t > 4
⎪ 0, otherwise
⎪
⎪⎩
b. x (t ) = e − t
u (t ) and h ( t ) = e −2 t
u (t )
14
⎨
(
∫
Solution:
∞ t
y (t ) = ∫ e −τ
u (τ ) e −2( t −τ )
u (t − τ ) dτ = ∫ e −τ
e − 2 ( t −τ )
dτ
−∞ 0
For t < 0 , there is no overlap and y (t ) = 0.
t
− t τ − t τ
t
− t t − t − t
y (t ) = e 2
∫ e dτ =e 2
e
0
= e 2
e0
− 1) =
e − e 2
, t ≥ 0
c. x (t ) = u ( − t ) an d h (t ) = δ ( t ) − 3e −2 t
u (t )
Solution:
y (t ) =
∞ ∞
h (τ ) x (t − τ )dτ = ⎡δ (τ ) − 3e −2τ
u (τ ) ⎤ u (τ − t ) dτ
∫ ∫ ⎣ ⎦
−∞ −∞
Now
∞
∫ δ (τ )u (τ − t ) dτ = u ( −t )
−∞
∞
3e −2τ ∞
3
For t < 0 , 3e −2τ
dτ = =
−2 2
0 0
∞ ∞
3e −2τ ∞
3
For t ≥ 0 , ∫ 3e −2τ
u (τ )u (τ − t )
dτ = ∫
3e −2τ
dτ = =
−2 2
e −2 t
−∞
⎧ 3
+ 1 =
5
,
y (t ) =
⎪ 2 2
3⎪ e −2 t
,
t t
t < 0
t ≥ 0
2
d. x (t ) = δ ( t − 2) + 3e 3 t
u ( −t ) and h (t ) = u (t ) − u (t − 1)
Solution:
15
1
⎜
e
1
⎜
e
⎩
y (t ) =
∞ ∞
h (τ ) x (t − τ )dτ = ⎡δ (t − τ − 2 ) + 3e 3( t −τ )
u ( − t + τ ) ⎤ [ u (τ ) − u (τ − 1) ] dτ
∫ ∫ ⎣ ⎦
−∞ −∞
∞ ∞
= ∫ δ (t − τ − 2) [ u (τ ) − u (τ − 1) ] dτ + ∫ 3e 3( t −τ )
u ( − t + τ ) [ u (τ ) − u (τ − 1) ]
dτ
−∞ −∞
Now
∞
∫ δ (t − τ − 2) [ u (τ ) − u (τ − 1) ] dτ = u (t − 2) − u ( t − 2 − 1) = Π ( t − 2.5 )
−∞
For t < 0 ,
∞ 1 1
∫ 3e 3( t −τ )
u ( − t + τ ) [ u (τ ) − u (τ − 1) ] dτ = ∫ 3e 3( t −τ )
dτ = 3e 3 t
∫ e − 3τ
dτ
−∞ 0 0
For 0 < t ≤ 1
,
= 3e
3 t
e −3τ
−3 0
= e 3 t ⎛
1
−
⎝
1 ⎞
3 ⎟
⎠
∞ 1 1
∫ 3e 3( t −τ )
u ( − t + τ ) [ u (τ ) − u (τ − 1) ] dτ = ∫ 3e 3( t −τ )
dτ = 3e 3 t
∫ e − 3τ
dτ
−∞ t t
⎧
e 3 t ⎛
1 −
1 ⎞
,
t ≤ 0
= 3e
3 t
e −3τ
−3 t
= e 3 t ⎛
e −3 t
−
⎝
1 ⎞
3 ⎟
⎠
⎪ ⎜
e 3 ⎟
⎪ ⎝ ⎠
y (t ) =
⎪
e 3 t ⎛
e −3 t
−
1 ⎞
, 0 < t ≤ 1⎨ ⎜
e 3 ⎟
⎪ ⎝ ⎠
⎪1, 2 ≤ t ≤ 3
⎪
0, ot herwi s e
2.10 The impulse response function of a continuous-time LTI is displayed in Fi gure P2.2(b). Assumi
ng the input x (t ) to the system is waveform illustrated in Figure P2.2(a), determine the syst em
output wavefor m y (t ) and sketch it.
Solution:
For t < 1 , there is no overlap
and
y (t ) = 0.
16
Figure P2.2
x (τ ) (a)
1
0 1 2 3
τ
4 5 6 7
h (t − τ )
1
(b)
1 < t < 2
τ
0 1 2 3 4
h (t − τ )
1
(c)
5 < t < 6
τ
0 1 2 3 4
As shown in Figure (b),
1
t -1
y (t ) = ∫ dτ = t
− 1
0
for 1 ≤ t < 2
For 2 ≤ t < 3
,
y (t ) = ∫ dτ
= 1
0
1
For 3 ≤ t < 4
,
y (t )
=
∫ dτ =1 − t + 3 = 4 − t
t − 3
Referring to Figure (c),
t
t
y (t ) = ∫ dτ
=
5
t −
5
for 5 ≤ t < 7
For 7 ≤ t < 8
,
y (t )
=
∫ dτ = t − t + 2 = 2
t − 2
7
For 8 ≤ t < 1 0
,
y
(
t
)
2
y (t ) =
17
∫ dτ = 7 − t + 3 = 10 − t
t − 3
1
1 2 3 4
t
5 6 7 8 9 10
C
C
2.11 An LTI system has the impulse response h (t ) = e −0.5 ( t − 2 )
u (t − 2) .
a. Is the system casual?
Solution:
Yes. The system is casual because h (t ) = 0 for t <
0 . b. Is the system stable?
Solution:
∞ ∞ −0.5 x
∞
The system is stable because
∫ h (t ) dt =
∫
e
e −0.5 x
dx = = 2
−0.5
−∞ 0 0
c. Repeat parts (a) and (b) for h (t ) = e −0.5 ( t + 2 )
u (t + 2) .
Solution:
The system is not causal but stable.
2.12 Wr ite down the exponential Fourier series coe fficients of the signal
x (t ) = 5 s in( 40π t ) + 7 cos ( 80π t − π / 2 ) − cos (160π t + π / 4 )
Solution:
Applying the Euler’s formula, we get the following terms:
⎡ e j 40 π t
− e − j 40 π t
⎤ ⎡ e j 80 π t
e − jπ / 2
+ e − j 80 π t
e jπ / 2
⎤ ⎡ e j 160 π t
e jπ / 4
+ e − j 160 π t
e − jπ / 4
⎤
x (t ) = 5 ⎢ ⎥ + 7 ⎢ ⎥ − ⎢ ⎥
⎣ 2 j ⎦ ⎣ 2 ⎦ ⎣ 2 ⎦
= − j 2.5e j 40 π t
+ j 2.5e − j 40 π t
− j 3.5e j 80 π t
+ j 3.5e − j 80π t
− 0.5e jπ / 4
e j 160 π t
− 0.5e − jπ / 4
e − j 160 π t
The Fourier coefficients are
1 = − j 2.5, C −1
=
j 2.5
2 = − j 3.5, C −2
=
j 3.5
C3 =
−
0.5e jπ /4
, C−3
= −0.5e − jπ /4
a. Is x (t ) period ic? If so, what is its peri od?
Solution:
18
19
Yes. ω = 40π ⇒ f = 20,
T
1 1
= . The period is = 0.05 sec .
o o o
20 20
2.13 A signal has the two-sided spectrum representation shown in Figure P2.3.
Figure P2.3
a. Wr ite the equation for x(t ).
Solution:
x (t ) = 14 cos
⎛
100π t −
π ⎞
+ 10 + 8 cos
⎛
300π t −
π ⎞
⎜
3
⎟ ⎜
2
⎟
⎝ ⎠ ⎝ ⎠
b. Is the signal periodic? If so, what is its period?
Solution:
Yes. It is periodic with fundame ntal period T =
1
= 0.02 .o
50
c. Does the signal have energy at DC?
Solution:
Yes as indicated by the presence of DC term C0 = 10 .
2.14 Wr ite down the complex exponential Fourier seri es for each of the periodic signals shown in
Figure P2.4. Use odd or even symme t ry whenever possible.
Solution:
a. T = 3, f = 1 / 3
o o
0
3
⎢ ∫ ∫ ⎥
3
3
⎣ ⎦
1
0 ⎢ ∫ ⎥
1 ⎡ ⎤ 1 ⎡ ⎤
⎣
− j
⎜ ⎟
n
,
1 ⎡ 2 3
⎤ 1
C = 2d t − 1d t = ( 4 − 1) = 1
⎣ 0 2 ⎦
1 ⎡ 2
C =
2 πnt
2e
− j
3
dt −
3 2 πnt
e
− j
3
dt
⎤
n ⎢ ∫ ∫ ⎥
⎣ 0 2 ⎦
j 3 ⎡ 2 πnt 2
− j
2 πnt 3
⎤
= ⎢ 2e 3
3 × 2π n ⎢⎣
− e 3
⎥
⎥⎦
0 2
j
= ⎡ 2e − j 4 π n / 3
− 2 − e − j 2 π n
+ e − j 4 π n / 3
⎤
2π n
j 3 3e − j 2 π n / 3
⎛ e + j 2 π n / 3
− e − j 2 π n / 3
⎞
= ( e − j 4 π n / 3
− 1) = ⎜ ⎟
2π n
3e −j2 πn/3
π n ⎝ 2 j ⎠
⎛ 2πn ⎞
= sin ,
π n ⎝ 3 ⎠
n = ± 1, ±2, .....
b. T = 2, f = 1 / 2
o o
1 ⎡ 1
⎤ 1 t 2
1
C = t dt =
2 2 2
=
4
(1 − 1) = 0
⎣ −1 ⎦ −1
1 1
C = ∫ t e − jπ nt
d t = − ∫ t d ( e − jπ n t
)n
2
⎢ ⎥
j 2π n
⎢ ⎥
⎣ −1 ⎦ ⎣ −1 ⎦
1
j ⎡ 1
= te − jπ n t
− e − jπ n t
d t
⎤
2π n
⎢ −1 ∫ ⎥
− 1 ⎦
j ⎡ 1 1 ⎤
= e − jπ n
+ e + jπ n
+ e − jπ n t
2π n
⎢
jπ n −1 ⎥
⎣ ⎦
j ⎡ e − jπ n
e + jπ n
e − jπ n
e jπ n
⎤
=
2
⎢
π n
+
π n
+
jπ 2
n 2
− jπ 2
n 2 ⎥
⎣ ⎦
je − jπ n
j ( − 1)= = n = ± 1, ± 2, .....
c. T
π n
= 2, f
π n
= 1 / 2
o o
2 A ⎡ 1
1
⎤ ⎛ t 2
⎞ ⎛ 1 ⎞ A
0C =
2
⎢ ∫ (1 − t )dt ⎥ = A ⎜ t −
2⎟
= A ⎜ 1 −
2
⎟ =
2
⎣ 0 ⎦ ⎝ ⎠ 0
⎝ ⎠
20
3
n
π n π n
1
⎥⎦
=
⎤
⎢
4
⎡ 1 ⎤
⎡ ⎤
⎢
0
3 ⎥
Since x (t ) is an even function of time,
A ⎡ 2 A
1
⎤ 1 1
C = n
=
2 2 ∫ (1 − t ) cos(π nt )dt ⎥ = A ∫ cos(π nt )dt − A ∫ t cos(π nt )dt
⎣ 0 ⎦ 0 0
1 1
A A ⎡ 1 ⎤
= 0 − ∫ td ( si n(π nt ) )dt = ⎢ − t si n(π nt ) 0
+ ∫ sin(π nt )dt ⎥
0 ⎣ 0 ⎦
A ⎡ co s (π nt ) ⎤ A A
= ⎢ 0 −
π n ⎢⎣ π n
⎥ = −
π 2
n 2 [ cos(π n ) − 1] =
π 2
n 2 [1 − cos(π n ) ]
⎧ 0,
⎪
⎨ 2A
,
n even
n odd
π 2
n 2
d. T = 3, f = 1 / 3
o o
1 3 / 2 2
1
2A ⎡
C = t dt+ 1dt
⎤
=
2A ⎡ t
+ t
3/ 2
=
1 ⎡ 1
+
1 ⎤
=
2A
0 ⎢ ∫ ∫ ⎥
⎣ 0 1 ⎦ 3
⎢
2⎣ 0
1
⎥
3 2 2 3⎦
Since x (t ) is an even function of time,
A 2 A
1.5
2 A
1 3 / 2
C = n
=
⎡
x (t ) cos( 2π nt / 3)dt
⎤
=
⎡
t cos( 2π nt / 3)dt
+
cos( 2π nt / 3)dt
⎤
n
2
⎢
3 ∫ 4 ⎥
3
⎢ ∫ ∫ ⎥
⎣ 0 ⎦ ⎣ 0 1 ⎦
Now
1 1 1
3 ⎡ ⎤ 3 ⎡ 1 ⎤
∫ t cos( 2π nt / 3)dt =
2π n
⎢ ∫ t d ( sin( 2π nt / 3) ) ⎥ =
2π n
⎢ t si n( 2π nt / 3) 0
− ∫
sin( 2π nt / 3)dt ⎥
0 ⎣ 0 ⎦ ⎣ 0 ⎦
3 3
= sin( 2π n / 3) + cos( 2π nt / 3)
2π n 2π n 0
3 3
= sin( 2π n / 3) + [ cos( 2π n / 3) − 1]
2π n 2π n
3/ 2
3 3/ 2 3
∫ cos( 2π nt / 3)dt = sin( 2π nt / 3) = − si n( 2π n / 3)
1
2π n 1
2π n
Substituting yields
21
2
5
T
Tn
⎦ ⎦
C
2A ⎛ 3 ⎞ ⎡ ⎛ 2πn ⎞ ⎤ 3A ⎡ ⎛ 2πn ⎞ ⎤
C = ⎜ ⎟ ⎢ cos ⎜ ⎟ − 1 = ⎢ cos ⎜ ⎟ − 1
n
3 ⎝ 2π n ⎠ ⎣ ⎝ 3 ⎠
⎥ 2π 2
n 2
⎣ ⎝ 3 ⎠
⎥
e. x (t ) =
where
∞
∑n
=−∞
p ( t − 8 n )
p ( t ) = Π ⎣⎡ ( t − 2
) / 2
+ Π ⎣⎡ ( t − 2 )
/ 4
− Π ⎡⎣ ( t − 6 )
/ 2
− Π ⎡⎣ ( t − 6 ) / 4
The FT of the pulse shape p ( t ) over [ 0,
To ]
is given by
To
P ( f ) = ∫ p (t ) e − j 2 π ft
d t
0
The FS coefficients of a periodic signal with basic pulse shape p ( t ) are gi ven by
1
To
n
= ∫o 0
p (t ) e − j 2 π nf o t
d t
Comparing yields
C =
1
o
Now
P ( f ) f = n f o
Π ⎡⎣ ( t − 2 ) /
2
Π ⎡⎣ ( t − 2 ) /
4 ⎤⎦Π ⎡⎣ ( t − 6
) / 2 ⎤⎦Π ⎡⎣ ( t −
6 ) / 4 ⎤⎦
←⎯ℑ
→ 2s i nc ( 2 f ) e − j 4 π f
←⎯ℑ
→ 4s i nc ( 4 f ) e − j 4 π f
←⎯ℑ
→ 2s i nc ( 2 f ) e − j 12 π f
←⎯ℑ
→ 4s i nc ( 4 f ) e − j 12 π f
⎣ ⎦ ⎣ ⎦
5
Therefore,
P ( f ) = 2si nc ( 2 f ) e − j 4 π f
⎡1 − e − j 8π f
⎤ + 4si nc ( 4 f ) e − j 4 π f
⎡1 − e − j 8π f
⎤
The FS coefficients of a periodic signal x (t ) are now obtained as
22
T
{ ⎣ ⎦ ⎣ ⎦
1
n
T
n
x (ν )e
− o o
dν e
j 2 π nf
ν
n
C
o T
C o
1
n = 2si nc ( 2 nf o
o
) e − j 4 π nf o
⎡1 − e − j 8π nf o
⎤ + 4si nc
( 4 nf
) e − j 4 π nf o
⎡1 − e − j 8 π nf o
⎤}
= {sinc ( n / 4 ) e − jπ n /2
⎡1 − e − jπ n
⎤ + 2sinc ( n / 2 ) e − jπ n /2
⎡1 − e − jπ n
⎤}4 ⎣ ⎦ ⎣ ⎦
2.15 For the rectangular pulse train in Figure 2.23, compute the Fourier coefficients of the new periodic
signal y( t) given by
a. y (t ) = x (t − 0.5To )
Solution:
The FS expansion of a periodic pulse train
∞
x (t )
⎡ ( t −nTo ) ⎤
of
= ∑ Π ⎢ ⎥
rectangular pul ses is given
by
n =−∞ ⎣ τ ⎦
x (t ) =
∞
∑n
=−∞
C x
e j 2 π nf o t
where the exponential FS coefficients are
C x
=
τ
sinc ( nf τ )
n o
o
Let the FS expansion of a periodic pulse train y (t ) = x (t − 0.5To ) be expressed as
y (t ) =
where
∞
∑n
=−∞
C y
e j 2 π nf o t
1
C y
= y (t ) e
− j 2 π n f o t
dt
=
1 x (t − 0.5T )e − j 2 π n f o t
dt
n
T ∫ T ∫ o
o To o To
1 j 2 π nf (ν + 0 . 5T ) − j 2 π n f o ( 0 .5 To ) 1
= ∫ = ∫ x (ν )e − o
dν
T To To
= e − jπ n
C x
o
x
n
Time Shifting introduces a linear phase shift in the FS coeffi cients;
their magnitudes are not changed.
23
j 2 π n f
t
∫ x (t ) e o
e o
dt
j 2 π tf ( n
−1)
=
⎞
n
∫
2
b. y (t ) = x (t ) e j 2 π t / To
Solution:
1
C y
=
1
∫ y (t ) e − o
dt = j 2 π t f − j 2 π nf t
n
T T o To o To
1
= x (t )e
− o
dt
To To
C x n
−1
c. y (t ) = x (α t )
Solution:
1 ( ) − π o
1 − π o
C y
= ∫ y t e j 2 n f t
dt = ∫ x ( α
t )e
j 2 n f t
dt
n
T T o To
− j π n
⎛ f o ν
o To
1
x ( )e
⎜
α
⎟
d
=
α T ∫ ν ⎝ ⎠
ν
= C x
o α To
Time Scaling does not change FS coefficients but the FS itself has changed as the
f 2 f 3 f
harmonic components are now at the frequencies ± o
, ± o
, ± o
, …
α α α
2.16 Draw the one-sided power spectrum for the square wave in Figure P2.5 with duty cycle 50%.
Figure P2.5
Solution:
The FS expansion for the square wave is given by
24
n
T
( − jπ n − j 2 π n − jπ n
)
=
One-sidedPowerSpect
rum
C
{
C π n
x (t ) =
where
∞
∑n
=−∞
C e j 2π nf o t
1
n =
T ∫ x (t ) e − j 2 π n f o t
dt
o To
To /2 To
1 ⎧⎪
=
T
⎨
⎫⎪
A ∫ e − j 2 π nf o t
dt − A ∫ e − j 2 π nf o t
dt ⎬
o 0 To / 2
A
e − j 2 π n f t To /2
e − j 2 π n f t o
=
T ( − j 2π nf )
o
− o
0
To / 2 }
o o
jA
= e − 1 − e + e
2π n
Therefore,
⎧
−
j2A
, n odd
⎪
n ⎨
⎩⎪ 0, ot herwise
Averag e power in the frequency component at
f
= nf equals C
2
. Figure
o n
displays the one-sided power spectrum for the square wave.
0. 9
One-s i ded P ower S pec t rum of S quare wave
0. 8
0. 7
0. 6
0. 5
0. 4
0. 3
0. 2
0. 1
0
0 5 10 15 20 25
F requenc y (x f )o
a. Calculate the normalized average power.
Solution:
25
n Total Power
including current
Fourier coefficient
1 0.8106
3 0.9006
5 0.9331
7 0.9496
9 0.9596
11 0.9663
13 0.9711
15 0.9747
17 0.9775
19 0.9798
21 0.9816
The norma lized average power for a periodic signal x (t ) is given by
To /2 2
1
P = ∫ x (t )
2
dt =
ATo
= A 2
x
T To − To /2 o
b. Determine the 98% power bandwidth of the pulse train.
Solution:
98% Power bandwidt h = 21 × f o
2.17 Determine the Fourier transforms of the signals shown in Figure P2.6.
Figure P2.6
26
1
1 ⎣
1
⎣ ⎦
2
2
2 ⎣
2
2 2 ⎣
Solution (a)
The pulse x (t ) can be expressed as
x (t ) = A ⎡ Π ( t + 0.5 ) − Π ( t − 0.5 )
Now
Π ( t ) ←⎯ℑ
→ sinc ( f )
Applying the time-shifting property of the FT, we obtain
Π ( t + 0.5 ) ←⎯ℑ
→ sinc ( f ) e jπ f
Π ( t − 0.5 ) ←⎯ℑ
→ sinc ( f ) e − jπ f
Adding
jπ f − jπ f jπ f − jπ f
X ( f ) = A ⎣⎡ s i nc ( f
) e
− sinc ( f )
e
= Asinc ( f ) ⎣⎡ e − e
= Aj 2sinc ( f ) s in ( π f
) =
j 2π f A ⎡ si nc 2
( f ) ⎤
Solution (b)
The pulse x (t ) can be expressed as
x (t ) = Π ( 2t ) co s ( 2π t )
Now
1
Π (2t ) ←⎯ℑ
→ s i nc ( f / 2)
2
cos( 2π t ) ←⎯ℑ
→
1
⎡δ ( f − 1) + δ ( f + 1)
X ( f ) = ℑ {Π (2t )} ⊗ ℑ {cos (2π t )} =
1
s i nc ( f / 2) ⊗
1
⎡δ ( f − 1) + δ ( f + 1)
1
=
4
⎡
⎣ si nc ⎡⎣ 0.5 ( f − 1)
3
+ si nc ⎡⎣ 0.5 ( f + 1) ⎤
⎦
Solution (c)
The pulse x (t ) can be expressed as
27
3
−
1
2 ⎣ ⎦
4
2
x (t ) = p ( t ) + p ( − t )
where
p ( t ) = e − t
⎡⎣ u ( t ) − u ( t − 1)
From Table 2.2, we have
1
e − t
u (t ) ←⎯ℑ
→
1 + j 2π f
Using the time-shifting property of the FT, we obtain
e − j 2 π f
e − t
u ( t − 1) = e −1
e − ( t −1)
u ( t − 1) ←⎯ℑ
→ e − 1
=
1 + j 2π f
e
− (1+ j 2 π f )
1 + j 2π f
Combining
1 e
− (1+ j 2 π f )
1
p (t ) ←⎯ℑ
→ P ( f ) = − = ⎡1 − e
− (1+ j 2 π f ) ⎤
1 + j 2π f 1 + j 2π
f
1 + j 2π f ⎣ ⎦
By time-reversal pr operty,
p ( −t ) ←⎯ℑ
→ P ( − f
) =
1
1 − j 2π
f
⎡1 e
− (1− j 2 π f ) ⎤
⎣ ⎦
X f = P f + P − f
1
= 1 −
e
1
+ 1 − e
( ) ( ) ( ) ⎡ − (1+ j 2 π f ) ⎤ ⎡ − (1− j 2 π f ) ⎤
3
1 + j 2π f ⎣ ⎦ 1 − j 2π f ⎣ ⎦
= {(1 − j 2π f ) ⎡1 − e
− (1+ j 2 π f ) ⎤ + (1 + j 2π f ) ⎡1 − e
− (1− j 2 π f ) ⎤}1 + ( 2π f )
⎣ ⎦ ⎣ ⎦
2
=
1 + ( 2π f )
⎡1 − e −1
cos ( 2π f ) + 2π f e −1
sin ( 2π f ) ⎤
Solution (d)
The pulse x (t ) can be expressed as
4x (t ) = Π ( t / 2 ) + Π ( t / 4 )
Now
28
4
W
c
2 ⎣ c c
⎣ ⎦ ⎣ ⎦
Π ( t / 2 ) ←⎯ℑ
→ 2s i nc ( 2 f )
Π ( t / 4 ) ←⎯ℑ
→ 4s i nc ( 4 f )
Adding
X ( f ) = 2s i n c ( 2 f ) + 4s i n c ( 4 f )
2.18 Use properties of the Fourier transform to compute the Four ier transform of
following signals.
a. sinc 2
(Wt )
Solution:
Λ (t / τ ) ←⎯ℑ
→
τ
s i nc 2 ⎛ fτ ⎞
2
⎜
2
⎟
⎝ ⎠
Using the duality property, we obtain
Wsinc 2
(Wt ) ←⎯ℑ
→Λ ( f / 2W )
sinc 2
(Wt ) ←⎯ℑ
→
1
Λ ( f / 2W )
Thus the Fourier transform of a sinc 2
pulse is a triangular function in frequency.
b. Π (t / T ) cos( 2π f c t )
Solution:
Π ( t / T ) ←⎯ℑ
→ T s i nc ( f T )
cos( 2π f t ) ←⎯ℑ
→
1
⎡δ ( f − f ) + δ ( f + f )
X ( f ) = ℑ Π t / T ⊗ ℑ co s ( 2π f t
)
= T s i
n c
1
f T ⊗ ⎡δ f − f + δ f + f ⎤
{ ( )} {
T
c } ( )
2 ⎣ ( c ) ( c ) ⎦
=
2
{sin c ⎡T ( f − f c ) ⎤ + si nc ⎡T ( f + f c ) ⎤}
c. ( e − t
cos 10π t ) u (t )
Solution:
29
2 2
⎢ ( + π 2
) + π − π 2 2
⎥
−
t
1
2
From Table 2.2,
1
e − t
u (t ) ←⎯ℑ
→
1 + j 2π f
Now
ℑ {( e − t
u (t ) ) cos 10π t } = ℑ {( e − t
u (t ) )} ⊗ ℑ {cos 10π t }
1 1
=
1 + j 2π f
⊗
2
⎡⎣δ ( f − 5 ) + δ ( f + 5 )
1 ⎡ 1 1 ⎤
= ⎢ + ⎥
2 ⎣ 1 + j 2π ( f − 5 ) 1 + j 2π ( f + 5 ) ⎦
1 ⎡ 1 + j 2π ( f + 5 ) + 1 + j 2π ( f − 5 ) ⎤
=
2
⎢
⎢⎣
⎥
(1 + j 2π f ) − ( j 2π 5 ) ⎥⎦
⎡ 1 + j 2π f ⎤
= ⎢ 2 2
⎥
(1 + j 2π f ) + 100π
⎡ 1 + j 2π f ⎤
d. te − t
u (t )
= ⎢ ⎥
1 100 j 4 f 4 f⎣ ⎦
Solution:
Applying the differentiation in frequency domain property in (2.85), we obtain
− t ℑ j d
te u (t ) ←⎯→
2π df
ℑ {e u (t )}
That is,
j d ⎡ (1 + j 2π f )
−
⎤
j 1− t ⎣ ⎦ℑ {te u (t )} =
2π df
1
= 2
= −
2π
j 2π
(1 + j 2π f )
(1 + j 2π f )
e. e− π t 2
Solution:
30
e − π t
e − j 2 π ft
dt e
− ( 2
)
dt
2
∞ ∞
2
2 2
⎜ ⎟
2 ⎣ ⎦
2
X ( f )
∞ ∞
2 π t + j 2 ft
= ∫ = ∫−∞ −∞
− π f 2
π f 2
Multiplying the ri ght hand side by e e yields
X ( f ) e − πf
e
− π ( t 2
+ j 2 f t − f 2
) dt e − π
f 2 e
− π ( t + jf )
dt
= ∫ = ∫−∞ −∞
Substituting t + j f = ν , we obtain
X ( f ) = e −
π f
∞
∫ e − πν
dν
−∞
1
f. 4si nc 2
(t ) cos (100π t )
Solution:
4si nc 2
( t ) ←⎯ℑ
→ 4 Λ
⎛ f ⎞
⎝ 2 ⎠
cos (100π t ) ←⎯ℑ
→
1
⎡δ ( f − 50 ) + δ ( f + 50 ) ⎤
X ( f ) = ℑ { 4sinc 2
( t )} ⊗ ℑ {cos (100π t )} = 4 Λ
⎛ f ⎞
⊗
1
⎡δ ( f − 50 ) + δ ( f + 50
) ⎤
= 2 { Λ ⎡⎣ 0.5 ( f −
50 )
⎜ ⎟
⎝ ⎠
+ Λ ⎡⎣ 0.5 ( f − 50 ) ⎤⎦}
2 ⎣ ⎦
2.19 Find the following convolutions:
a. sinc (Wt ) ⊗ si nc ( 2Wt )
Solution:
We use the convolution property of Fourier transfor m in (2.79).
x (t ) ⊗ y (t ) ←⎯ℑ
→ X ( f )Y ( f )
Now
1
sinc ( 2Wt ) ←⎯ℑ
→ Π ( f / 2W )
2W
31
⎛ ⎞
55 ⎝ ⎠
Therefore,
1 1
sinc (Wt ) ⊗ s i nc ( 2Wt ) ←⎯ℑ
→ Π ( f / W ) × Π ( f / 2W )
W 2W
1
=
2W 2
Π ( f / W )
b. sinc 2
(Wt ) ⊗ sinc ( 2Wt )
Solution:
Again using the convolution property of Fourier transform
sinc 2
(Wt ) ⊗ si nc ( 2Wt ) ←⎯ℑ
→
1
Λ ( f / 2W ) ×
1
Π ( f / 2W )W 2W
1
=
2W 2
Λ ( f / 2W )
2.20 The FT of a signal
1
X ( f ) =
5 + j 2π f
x (t ) is described by
Determine the FT V ( f ) of the following signals:
a. v (t ) = x (5t − 1)
Solution:
v (t ) = x (5t − 1) = x
⎡
5
⎛
t −
1 ⎞ ⎤
⎢ ⎜
5
⎟ ⎥
⎣ ⎝ ⎠ ⎦
Let
1 f 1 1
y (t ) = x (5t ) ←⎯ℑ
→ Y ( f ) = X =⎜ ⎟
5 ( j 2π f / 5 ) + 5
1
=
j 2π f + 25
v (t ) = y
⎛
t
−
1 ⎞
←⎯ℑ
→ V ( f ) = Y ( f )
e
−j2 πf
5
=
1
−j2 πf
e 5
⎜
5
⎟
j 2π f + 2 5⎝ ⎠
32
⎡ + ⎤
⎡ + ⎤
⎟
b. v (t ) = x (t ) cos(100π t )
Solution:
x (t ) e j 100 π t
x (t ) e − j
100 π t
v (t ) = x (t ) cos(100π t ) = ⎣ ⎦
2
Now
x (t ) e j 100 π t
x (t ) e − j
100 π t
⎣ ⎦
2
1
←⎯ℑ
→ V ( f ) =
2
X ( f − 50 ) + X ( f + 50 ) ⎤⎦
1 1
=
j 2π ( f − 50 ) + 5
+
j 2π ( f + 50 ) + 5
After simplification, we get
j2πf +5
V ( f ) =
j 20π f + ( π 2
1 0 4
+ 25 − 4π 2
f 2
)
c. v (t ) = x (t ) e j 10 t
Solution:
v (t ) = x (t ) e j 10 t
←⎯ℑ
→ V ( f ) = X
⎛
f −
5 ⎞
=
1
⎜ ⎟
⎛ ⎞⎝ π ⎠ j 2π ⎜ 5
f − + 5
π⎝ ⎠
dx (t)
d. v (t ) =
dt
Solution:
Using the differentiation property of FT
d
x (t ) ←⎯ℑ
→ j 2πfX ( f ) , we obtain
dt
V ( f ) =
j2πf
j 2π f X ( f ) =
5 + j 2π f
e . v (t ) = x (t ) ⊗ u (t )
Solution:
Using the convolution property of FT x (t ) ⊗ y (t ) ←⎯ℑ
→ X ( f )Y ( f ) , we can write
33
1 ⎛ 1 1 ⎞
V ( f ) = X ( f )U ( f ) = δ ( f ) +
5 + j 2π f
⎜
2 j 2π f
⎟
⎝ ⎠
1 1 1
=
2
δ ( f )
5 + j 2π f
+
j 2π f ( 5 + j 2π f )
1 1
= δ ( f ) +
10 j 2π f ( 5 + j 2π f )
2.21 Consider the delay eleme nt y (t ) = x (t −
3) . a. What is the impulse response h( t)?
Solution:
Si nc e y (t ) = h (t ) ⊗ x (t ) = x (t − 3) , the impulse response of the delay eleme nt is
given from ( 2.16) as
h (t ) = δ (t − 3)
b. What is the ma gnitude and phase response function of the system?
Solution:
H ( f ) = ℑ {δ (t − 3)} = e − j 6 π f
H ( f ) = 1
(H ( f ) = − 6π f
2.22 The periodic input x (t ) to an LTI system is displaye d in Figure P2.7. The
frequency response function of the system is given by
2
H ( f ) =
2 + j 2π f
Figure P2.7
34
o
T
o o
T
To
Cn
o
a. Wr ite the complex exponential FS of input x (t ) .
Solution:
The input x (t ) is rectangular pulse train in Example 2.24 shifted by
T
/ 4 and
duty cycle
τ
= 0.5 . That is,
o
x (t ) g ( t T
∞
/ 4 )
⎡ ( t −nTo
− To
/4) ⎤
= To
− o
= ∑ Π ⎢
0.5T
⎥
n =−∞ ⎣ o ⎦
The comple x exponential FS of
τ
g (t ) from Exa mple 2.24 withTo
T = 2 ( f = 0.5 ) and = 0.5 is given by
o
∞
g (t ) = 0.5 ∑ s i nc ( 0.5 n )e jπ nt
n
=−∞
In Exercise 2.15(a), we showed that time shifting introduces a linear phase shift
in the FS coefficients; their ma gnitudes are not changed. The phase shift is
equal
to e − j 2 π nf o u
for a time shift of u. The exponential FS
coefficients
x (t ) are
= 0.5s i nc ( 0.5n ) e
− j 2 π nf o ( 0. 2 5To )
= e − jπ n /2
0.5s inc ( 0.5n )
∞
x (t ) = 0.5 ⎡ e − jπ n /2
s i nc ( 0.5 n ) ⎤e jπ nt
∑ ⎣ ⎦
n =−∞
b. Plot the ma gnitude and phase response functions for H ( f ) .
Solution:
H ( f ) =
2
=
1
2 + j 2π f 1 + jπ f
1 1
H ( nf o ) =
1 +
jπ f
=
1 + jπ nf
35
0 2 4 6 8 0 2 4 6 8
F r equenc y (f) F r eq uenc y (
f)
n
=−∞
n = −∞ ⎢ ⎥
∑ ⎢ ⎥
MagnitudeRespons
e(dB)
PhaseRespons
e(degrees)
0 0
-1 0
-5
-2 0
-1 0 -3 0
-1 5
-4 0
-5 0
-2 0 -6 0
-2 5
-7 0
-8 0
-3 0 -9 0
c. Compute the complex exponential FS of the output y (t ) .
Solution:
Using (2.114), the output of an LTI system to the input
x (t ) =
∞
⎡ 0.5e − jπ n /2
s i nc ( 0.5 n ) ⎤e jπ nt
∑ ⎦
Cn
is given by
⎡ ⎤
∞ ∞ ⎢ 0.5e − jπ n /2 ⎥
y (t ) = ∑ C n H (
0.5n ) e
n =−∞
jπ n t
= s inc ( 0.5n ) e jπ n t
1 + j 05π n
FS co effi cien t o f y (
t )
2.23 The frequency response of an ideal LP filter is given by
⎧⎪5e − j 0.0025π f
,
H ( f ) = ⎨
f < 1000 Hz
0, f > 1000 Hz
Determine the output signal in each of the followi ng cases:
a. x (t ) = 5 sin ( 400π t ) + 2 cos (1200π t − π / 2 ) − cos ( 2200π t + π / 4 )
36
⎝ ⎠
⎜
2200
⎟ ⎜
2000
⎟ ⎜
2000
⎟
Solution:
5, f < 1000 Hz
H ( f ) = ⎨
⎪⎩
⎧⎪
0,
−0.0025π f ,
f > 1000 Hz
f < 1000 Hz
(H ( f ) = ⎨
⎪⎩ 0, f > 1000 Hz
Since H ( 200) = 5 and (H ( 200) = −π / 2 , the output of the system for an input
5sin(400πt ) can now be expressed using (2.117) as 25sin(400πt − π/2).
Next H ( 600) = 5 and (H ( 600) = −3π / 2 = π / 2 , the output of the system for
an input 2 cos (1200π t − π / 2 ) can now be expressed using (2.117) as
10 cos (1200π t − π / 2 + π / 2 ) = 10 cos (1200π t ) .
Now H (1100) = 0. So the LP filter doesn’t pass cos ( 2200π t + π / 4 ) . The
output of the LP filter is therefore given by
y (t ) = 25 sin ( 400π t − π / 2 ) + 10 cos (1200π t )
sin( 2200π t
)b. x (t ) = 2 si n( 400π t ) +
π t
Solution:
Since H ( 200) = 5 and (H ( 200) = −π / 2 , the output of the system for an input
2sin(400πt ) is 10sin(400πt − π/2).
To calculate the response to
sin( 2200π t)
, we note that
π t
sin( 2200π t) ⎛ f ⎞= 2200 × sinc ( 2200t ) ←⎯ℑ
→Π
π t
⎜
2200
⎟
In fr equency domain, the output of LP filter to
sin( 2200π t)
is
π t
Π
⎛ f ⎞
5e − j 0.0025 π f
Π
⎛ f ⎞
= 5Π
⎛ f ⎞
e − j 0.0025π f
⎝ ⎠ ⎝ ⎠ ⎝ ⎠
37
⎜ ⎟
⎝ ⎠
⎜
1000
⎟ ⎜
1000
⎟
⎜ ⎟
⎣ ⎦
Now
Π
⎛ f ⎞
e − j 0.0025 π f
←⎯ℑ
→ 2000sinc
⎝ 2000 ⎠
2000 ( t − 0.00125 )
The output of the LP filter is therefore given by
y (t ) = 10 sin ( 400π t − π / 2 ) +
10000si nc
sin(1000π t)c. x (t ) = cos( 400π t ) +
π t
2000 ( t − 0.00125 )
Solution:
Since H ( 200) = 5 and (H ( 200) = −π / 2 , the output of the system for an input
cos( 400π t ) is 5 cos ( 400π t − π / 2 ) .
To calculate the response to
sin(1000πt)
, we note that
π t
sin(1000π t) ⎛ f ⎞= 1000 × si nc (1000t ) ←⎯ℑ
→Π
π t
⎜
1000
⎟
In fr equency domain, the output of LP filter to
5e − j 0.0025π f
Π
⎛ f ⎞
= 5Π
⎛ f ⎞
e − j 0.0025π f
⎝ ⎠ ⎝ ⎠
Now
sin(1000πt)
is
π t
Π
⎛ f ⎞
e − j 0.0025 π f
←⎯ℑ
→ 1000sinc ⎡1000 ( t − 0.00125 ) ⎤
⎝ 1000 ⎠
⎣ ⎦
The output of the LP filter is therefore given by
y (t ) = 5 cos ( 400π t − π / 2 ) + 5000si nc ⎡1000 ( t − 0.00125 ) ⎤
d. x (t ) = 5 cos(800π t ) + 2δ (t )
Solution:
38
⎨
4,
Since H ( 400) = 5 and (H ( 4 00) = −π , the output of the system for an input
5 cos ( 800π t ) is 25 cos ( 800π t − π ) . The response of the system to δ (t ) is h (t ) .
The impulse response of the ideal LP filter is 10000sinc
Co mbining
2000 ( t − 0.00125 ) ⎤⎦ .
y (t ) = 25 cos ( 800π t − π ) + 2 h (t
)
= 25 cos ( 800π t − π ) + 20 ×
103
sinc
2000 ( t − 0.00125 )
2.24 The frequency response of an ideal HP filter is given by
H ( f ) =
⎧⎪
⎪⎩ 0,
f > 20 Hz ,
f < 20 Hz
Determine the output signal y (t ) for the input
a. x (t ) = 5 + 2 cos ( 5 0π t − π / 2 ) − cos ( 75π t + π / 4 )
Solution:
y (t ) = 8 cos ( 50π t − π / 2 ) − 4 cos ( 75π t + π / 4 )
b. x (t ) = cos ( 2 0π t − 3π / 4 ) + 3 cos (100π t + π / 4 )
Solution:
y (t ) = 12 cos (100π t + π / 4 )
2.25 The frequency response of an ideal BP filter is gi ven by
H ( f ) =
⎧⎪
⎨
⎪⎩
2e − j 0.0005 π f
, 900< f
0, otherwise
< 1000 Hz ,
Determine the output signal y (t ) for the input
a. x (t ) = 2 cos (1850π t − π / 2 ) − cos (1900π t + π / 4 )
Solution:
H (925) = 2 and H (950) = 2 .
39
⎝ ⎠
⎡ ⎛ − ⎞ ⎛ + ⎞ ⎤
⎛ − ⎞
⎛ − ⎞
⎛ + ⎞
(H ( f ) = − 0.0005π f . Therefore,
(H (925) = − 0.0005π
f
(H (950) = −0.0005π f
= −0.0005π × 925 = − 0.462π
= − 0.0005π × 950 = − 0.475π
y (t ) = 4 cos (1850π t − π / 2 − 0.462π ) − 2 cos (1900π t + π / 4 − 0.475π )
b. x (t ) = si nc ( 60t ) cos (1900π t )
Solution:
1 ⎛ f ⎞ 1
X ( f ) =
60
Π ⎜
60
⎟ ⊗
2
[δ ( f − 950) + δ ( f +
950) ]
1 ⎡ ⎛ f −950 ⎞ ⎛ f +950 ⎞ ⎤
= Π + Π
120 ⎢ ⎜
60
⎟ ⎜
60
⎟ ⎥
⎣ ⎝ ⎠ ⎝ ⎠ ⎦
Y ( f ) = H ( f ) X ( f )
1 f 950 f 950
= 2 e − j 0. 0005 π f
× × Π + Π
120 ⎢ ⎜
60
⎟ ⎜
60
⎟ ⎥
⎣ ⎝ ⎠ ⎝ ⎠ ⎦
Now
1 f 950
×Π ←⎯→ s i nc 60t e
⎜ ⎟
ℑ
( )
j 1900 π t
60 ⎝ 60 ⎠
1 f 950
e ×Π ←⎯→ si nc ⎡ 60 t − 0.00025 ⎤ e
− j 0.0005π f ℑ
⎜ ⎟ ( ) j 1900 π ( t − 0.00025 )
60 ⎝ 60 ⎠
⎣ ⎦
Similarly
1 f 950
×Π ←⎯→ s i nc 60t e
⎜ ⎟
ℑ
( )
− j 1900 π t
60 ⎝ 60 ⎠
⎛ + ⎞
⎣ ⎦ ⎣ ⎦
⎣
1 f 950
e ×Π ←⎯→ si nc ⎡ 60 t − 0.00025 ⎤ e
− j 0.0005π f ℑ
⎜ ⎟ ( ) − j 1900 π ( t − 0.00025 )
60 ⎝ 60 ⎠
⎣ ⎦
Therefore,
y (t ) = sinc ⎡ 60 ( t − 0.00025 ) ⎤ ⎡ e
j 1900 π ( t − 0.00025 )
+ e
− j 1900 π ( t − 0.00025 ) ⎤
= 2s inc ⎡⎣ 60 ( t −
0.00025 )
cos ⎡1900π ( t − 0.00025 )
40
⎝ ⎠
⎡ ⎛ − ⎞ ⎛ + ⎞ ⎤
⎛ + ⎞
⎛ + ⎞
⎣ ⎦ ⎣ ⎦
c. x (t ) = si nc 2
( 30t ) cos (1900π t )
Solution:
1 ⎛ f ⎞ 1
X ( f ) =
30
Λ ⎜
60
⎟ ⊗
2
[δ ( f − 950) + δ ( f +
950) ]
1 ⎡ ⎛ f −950 ⎞ ⎛ f +950 ⎞ ⎤
= Λ + Λ
60 ⎢ ⎜
60
⎟ ⎜
60
⎟ ⎥
⎣ ⎝ ⎠ ⎝ ⎠ ⎦
Y ( f ) = H ( f ) X ( f )
1 f 950 f 950
= 2 e − j 0. 0005π f
× Λ + Λ
60 ⎢ ⎜
60
⎟ ⎜
60
⎟ ⎥
⎣ ⎝ ⎠ ⎝ ⎠ ⎦
Now
1 ⎛ f −950 ⎞
←⎯→ sinc 30t e
×Λ ⎜ ⎟
ℑ 2
( ) j 1900 π t
30 ⎝
e
60
1
⎠
⎛ f −950 ⎞
←⎯→ sinc ⎡ 30 t − 0.00025 ⎤ e
− j 0.0005π f
×Λ ⎜ ⎟
ℑ 2
( ) j 1900 π ( t − 0.00025 )
30 ⎝ 60 ⎠
⎣ ⎦
Similarly
1 f 950
×Π ←⎯→ s i nc 30t e
⎜ ⎟
ℑ 2
( )
− j 1 90 0 π t
30 ⎝ 60 ⎠
1 f 950
e ×Π ←⎯→ si nc ⎡ 30 t − 0.00025 ⎤ e
− j 0.0005π f
⎜ ⎟
ℑ 2
( ) − j 1900 π ( t − 0.00025 )
30 ⎝ 60 ⎠
⎣ ⎦
Therefore,
y (t ) = sinc 2
⎡ 30 ( t − 0.00025 ) ⎤ ⎡ e
j 1900 π ( t − 0. 00025 )
+ e
− j 1900 π ( t − 0. 00025 ) ⎤
⎣= 2s inc 2
⎡⎣ 30 ( t −
0.00025 )
cos ⎡1900π ( t − 0.00025 )
2.26 The signal 2e −2 t
u (t ) is input to an ideal LP filter with passband edge frequency
equal to 5 Hz . Find the energy density spectrum of the output of the filter. Calculat
e the energy of the input signal and the output signal.
Solution:
41
⎜
10
⎟
10
y
2 2
⎝ ⎠
−1
π 0
∞
E = 2e −2 t
u (t )
2
dt =
∞
4e −4 t
dt =4
e −4 t
∞
= 1
x ∫ ∫ −4−∞ 0 0
2
x (t ) = 2e −2 t
u (t ) ←⎯ℑ
→
2 + j 2π
f
The energy density spectrum of the output y( t ), Y ( f )
2
, is related to the energy
density spectrum of the input x (t )
Y ( f )
2
= H ( f )
2
X ( f )
2
= Π
⎛ f ⎞
⎝ ⎠
2
2 + j 2π f
∞ ∞ ∞ 2 2
E = y (t )
2
dt = Y ( f )
2
df = Π
⎛ f ⎞ 2
df
y ∫ ∫ ∫−∞ −∞ −∞
⎜ ⎟
2 + j 2π f
5
1
= ∫ 1 + jπ
f
2 5
df = 2
∫
1
df
1 + π 2
f 2
−5 0
Making change of variables π f = u ⇒ d
f
1
= d u , we obtain
π
2
5π
1 2 5π2
E = ∫0
du = tan
1 + u 2
π
( u ) = (1.507 ) = 0.9594
π
Thus output of the LP filter cont ains 96% of the input signal energy.
2.27 Calculate and sketch the power spectral density of the following signals. Calculate
the normalized average power of the signal in each case.
a. x (t ) = 2 cos (1000π t − π / 2 ) − cos (1850π t + π / 4 )
Solution:
x1 ( t ) x2 ( t )
1
R (τ ) = li m
T /2
x (t ) x (t − τ ) d t= R (τ ) +R (τ ) −R (τ ) −R (τ )
x
T →∞ T ∫− T /2
x1 x2 x1 x2 x2 x1
Now
42
⎣ ⎦
2 2
2
T
2
1
R (τ ) = li m
T /2
x (t ) x (t − τ ) dt
x1 T →∞ T
1
∫ 1 1
− T /2
T /2
⎛ π ⎞ ⎡ π ⎤
= li m
T ∫ 2 cos ⎜ 1000π t −
2
⎟ 2 cos 1000π ( t − τ ) −
2
d t
T →∞ ⎢ ⎥
− T /2 ⎝ ⎠ ⎣ ⎦
T /2
1
= 2 lim
T →∞ T ∫
− T /2
{cos (1000πτ ) + cos ⎡1000π ( 2t − τ ) − π ⎤} d t
= 2 cos (1000πτ )
The second term is zero because it integrates a sinusoidal func tion over
a period. Similarly,
1
Rx (τ ) = li m
T
T /2
∫x (t ) x (t − τ ) dt =
1
cos (1850πτ )
2 T →∞
− T /2
The cross-correlation term
1
Rx x (τ ) = li m
T
T /2
x (t ) x (t − τ ) = lim
1
1 2
T
T /2
2 cos (1000π t − π / 2 ) cos ⎡1850π ( t − τ ) + π/ 4 ⎤dt
1 2 T →∞
∫
− T /2
1
T /2
T →∞
∫ ⎣ ⎦
− T /2
= li m
T →∞ ∫
− T /2
{cos ( 850π t − 1850πτ + 3π / 4 ) + s in ( 2850π t − 1850πτ − π /
4 )}dt
is zero because it integrates a si nusoi dal function over a period in each
case. Similarly, it can be shown that all ot he r cross-correlation terms are
zero. Therefore,
1
Rx (τ ) = Rx1
(τ ) + Rx2
(τ ) = 2 cos (1000πτ ) +
2
cos (1850πτ )
⎧ 1 ⎫Gx ( f ) = ℑ { Rx (τ )} = ℑ ⎨ 2 cos (1000πτ ) + cos (1850πτ ) ⎬
⎩ ⎭
1
= [δ ( f − 500) + δ ( f + 500) ] +
4
[δ ( f − 925) + δ ( f + 925) ]
The norma lized average power is obtained by using (2.172) as
43
x ∫ x
PowerSpectralDens
ity
∞ ∞
1
∞
P = G ( f )df = ∫ [δ ( f − 500) + δ ( f + 500) ] df +
4 ∫ [δ ( f − 925) + δ ( f + 925)
] df
−∞ −∞ −∞
1 1 5
= 1 + 1 +
4
+
4
=
2
1
0. 8
0. 6
0. 4
0. 2
0
-1 -0. 8 -0. 6 -0. 4 -0. 2 0 0. 2 0. 4 0. 6 0. 8 1
F requenc y (k Hz
)
b. x (t ) = [1 + sin( 200π t ) ] cos( 2000π t )
Solution:
x (t ) = [1 + s in( 200π t ) ] cos( 2000π t )
= cos( 2000π t ) + s in( 200π t ) cos( 2000π t )
1 1
= cos( 2000π t ) +
2
sin ( 2200π t ) −
2
sin (1800π t )
x1 ( t )
Now
x2 ( t ) x3 ( t )
1
R (τ ) = li m
T /2
x (t ) x (t − τ ) d t= R (τ ) +R (τ ) +R (τ )
x
T →∞ T ∫− T /2
x1 x2 x3
where
1
Rx1
(τ ) =
2
cos ( 2000πτ )
1
Rx2
(τ ) =
8
cos ( 2200πτ )
1
Rx3
(τ ) =
8
cos (1800πτ )
44
⎩ ⎭
x ∫ x
PowerSpectralDens
ity
Therefore,
1 1 1
Rx (τ ) =
2
cos ( 2000πτ ) +
8
cos ( 2200πτ ) +
8
cos (1800πτ )
⎧ 1 1 1 ⎫
Gx ( f ) = ℑ { Rx (τ )} = ℑ ⎨
2
cos ( 2000πτ ) +
8
cos ( 2200πτ ) +
8
cos (1800πτ ) ⎬
1 1
=
4
[δ ( f − 1000) + δ ( f + 1000) ] +
16
[δ ( f − 1100) + δ ( f + 1100) ]
1
+
16
[δ ( f − 900) + δ ( f + 900) ]
The normalized average power is obtained by using (2.172) as
∞
1
∞
1
∞
P = G ( f )df =
4 ∫ [δ ( f − 1000) + δ ( f + 1000) ] df +
16 ∫ [δ ( f − 1100) + δ ( f +
1100) ] df
−∞ −∞ −∞
1
∞
1 1 1 3
+
16 ∫ [δ ( f − 900) + δ ( f +
900) ] d f
−∞
=
2
+
8
+
8
=
4
0. 25
0. 2
0. 15
0. 1
0. 05
0
-1 -0. 8 -0. 6 -0. 4 -0. 2 0 0. 2 0. 4 0. 6 0. 8 1
Frequenc y (k Hz )
c. x (t ) = cos 2
( 200π t ) s i n (1800π t )
Solution:
45
2 ⎣ ⎦ 2 2
1 2 3
⎩ ⎭
x ∫ x
x (t ) =
1
⎡1 + cos ( 400π t ) ⎤ sin (1800π t ) =
1
si n (1800π t ) +
1
sin (1800π t ) cos (
400π t )
1 1 1
=
2
sin (1800π t ) +
4
s i n (1400π t ) +
4
si n ( 2200π t )
Now
Rx (τ ) = Rx(τ ) +
Rx
(τ ) +
Rx
(τ )
where
1
Rx1
(τ ) =
8
cos (1800πτ )
1
Rx2
(τ ) =
32
cos (1400πτ )
1
Rx3
(τ ) =
32
cos ( 2200πτ )
Therefore,
1 1 1
Rx (τ ) =
8
cos (1800πτ ) +
32
cos (1400πτ ) +
32
cos ( 2200πτ )
⎧ 1 1 1 ⎫
Gx ( f ) = ℑ { Rx (τ )} = ℑ ⎨
8
cos (1800πτ ) +
3 2
cos (1400πτ ) +
3 2
cos ( 2200πτ ) ⎬
1 1
=
16
[δ ( f − 900) + δ ( f + 900) ] +
64
[δ ( f − 700) + δ ( f + 700) ]
1
+
64
[δ ( f − 1100) + δ ( f + 1100) ]
The normalized average power is obtained by using (2.172) as
∞
1
∞
1
∞
P = G ( f )df =
16 ∫ [δ ( f − 900) + δ ( f + 900) ]df +
64 ∫ [δ ( f − 700) + δ ( f +
700) ] df
−∞ −∞ −∞
1
∞
1 1 1 3
+
64 ∫ [δ ( f − 1100) + δ ( f +
1100) ] df
−∞
=
8
+
32
+
32
=
16
46
PowerSpectralDensi
ty
0. 1
0. 09
0. 08
0. 07
0. 06
0. 05
0. 04
0. 03
0. 02
0. 01
0
-1 -0. 8 -0. 6 -0. 4 -0. 2 0 0. 2 0. 4 0. 6 0. 8 1
Contemporary Communication Systems 1st Edition Mesiya Solutions Manual
Download:http://testbanklive.com/download/contemporary-communication-systems-
1st-edition-mesiya-solutions-manual/
contemporary communication systems mesiya pdf download
contemporary communication systems mesiya download
contemporary communication systems pdf
contemporary communication systems mesiya solutions
47

More Related Content

What's hot

Optical fibers Theory and applications
Optical fibers Theory and applicationsOptical fibers Theory and applications
Optical fibers Theory and applicationsPraveen Vaidya
 
Magnetostatics (1)
Magnetostatics (1)Magnetostatics (1)
Magnetostatics (1)Kumar
 
Digital signal Processing all matlab code with Lab report
Digital signal Processing all matlab code with Lab report Digital signal Processing all matlab code with Lab report
Digital signal Processing all matlab code with Lab report Alamgir Hossain
 
Differential stripline impedance calculate
Differential stripline impedance calculateDifferential stripline impedance calculate
Differential stripline impedance calculatePei-Che Chang
 
Nonequilibrium Excess Carriers in Semiconductors
Nonequilibrium Excess Carriers in SemiconductorsNonequilibrium Excess Carriers in Semiconductors
Nonequilibrium Excess Carriers in Semiconductorstedoado
 
EC6602-Antenna fundamentals new
EC6602-Antenna fundamentals new EC6602-Antenna fundamentals new
EC6602-Antenna fundamentals new krishnamrm
 
Ppt on sawtooth wave form generator
Ppt on sawtooth wave form generatorPpt on sawtooth wave form generator
Ppt on sawtooth wave form generatorAmit kumar
 
Fast Fourier Transform
Fast Fourier TransformFast Fourier Transform
Fast Fourier Transformop205
 
Common Emitter Configuration | Electronical Engineering
Common Emitter Configuration | Electronical EngineeringCommon Emitter Configuration | Electronical Engineering
Common Emitter Configuration | Electronical EngineeringTransweb Global Inc
 
Solved problems in waveguides
Solved problems in waveguidesSolved problems in waveguides
Solved problems in waveguidessubhashinivec
 
Photonic crystal fibers
Photonic crystal fibersPhotonic crystal fibers
Photonic crystal fibersajay singh
 

What's hot (20)

Optical fibers Theory and applications
Optical fibers Theory and applicationsOptical fibers Theory and applications
Optical fibers Theory and applications
 
Magnetostatics (1)
Magnetostatics (1)Magnetostatics (1)
Magnetostatics (1)
 
Digital signal Processing all matlab code with Lab report
Digital signal Processing all matlab code with Lab report Digital signal Processing all matlab code with Lab report
Digital signal Processing all matlab code with Lab report
 
Differential stripline impedance calculate
Differential stripline impedance calculateDifferential stripline impedance calculate
Differential stripline impedance calculate
 
Ppt
PptPpt
Ppt
 
Radix-2 DIT FFT
Radix-2 DIT FFT Radix-2 DIT FFT
Radix-2 DIT FFT
 
Nodal analysis
Nodal analysisNodal analysis
Nodal analysis
 
Optical receivers
Optical receiversOptical receivers
Optical receivers
 
Nonequilibrium Excess Carriers in Semiconductors
Nonequilibrium Excess Carriers in SemiconductorsNonequilibrium Excess Carriers in Semiconductors
Nonequilibrium Excess Carriers in Semiconductors
 
EC6602-Antenna fundamentals new
EC6602-Antenna fundamentals new EC6602-Antenna fundamentals new
EC6602-Antenna fundamentals new
 
Presentation on JFET
Presentation on JFETPresentation on JFET
Presentation on JFET
 
Ppt on sawtooth wave form generator
Ppt on sawtooth wave form generatorPpt on sawtooth wave form generator
Ppt on sawtooth wave form generator
 
8086 pin function
8086 pin function8086 pin function
8086 pin function
 
Fast Fourier Transform
Fast Fourier TransformFast Fourier Transform
Fast Fourier Transform
 
Lecture 5: The Convolution Sum
Lecture 5: The Convolution SumLecture 5: The Convolution Sum
Lecture 5: The Convolution Sum
 
Common Emitter Configuration | Electronical Engineering
Common Emitter Configuration | Electronical EngineeringCommon Emitter Configuration | Electronical Engineering
Common Emitter Configuration | Electronical Engineering
 
Solved problems in waveguides
Solved problems in waveguidesSolved problems in waveguides
Solved problems in waveguides
 
Modes
ModesModes
Modes
 
Photonic crystal fibers
Photonic crystal fibersPhotonic crystal fibers
Photonic crystal fibers
 
Timing diagram of microprocessor 8085
Timing diagram of microprocessor 8085Timing diagram of microprocessor 8085
Timing diagram of microprocessor 8085
 

Similar to Contemporary Communication Systems Solutions Manual Chapter 2

Mathematical formula tables
Mathematical formula tablesMathematical formula tables
Mathematical formula tablesSaravana Selvan
 
Operations on Continuous time Signals.
Operations on Continuous time Signals.Operations on Continuous time Signals.
Operations on Continuous time Signals.Shanawaz Ahamed
 
233_Sample-Chapter (1).pdf
233_Sample-Chapter (1).pdf233_Sample-Chapter (1).pdf
233_Sample-Chapter (1).pdfssuser4dafea
 
Signal and Systems part i
Signal and Systems part iSignal and Systems part i
Signal and Systems part iPatrickMumba7
 
8fbf4451c622e6efbcf7452222d21ea5_MITRES_6_007S11_hw02.pdf
8fbf4451c622e6efbcf7452222d21ea5_MITRES_6_007S11_hw02.pdf8fbf4451c622e6efbcf7452222d21ea5_MITRES_6_007S11_hw02.pdf
8fbf4451c622e6efbcf7452222d21ea5_MITRES_6_007S11_hw02.pdfTsegaTeklewold1
 
University of manchester mathematical formula tables
University of manchester mathematical formula tablesUniversity of manchester mathematical formula tables
University of manchester mathematical formula tablesGaurav Vasani
 
Master theorm practive problems with solutions
Master theorm practive problems with solutionsMaster theorm practive problems with solutions
Master theorm practive problems with solutionsKumar
 
Continuous Time Convolution1. Solve the following for y(t.docx
Continuous Time Convolution1. Solve the following for y(t.docxContinuous Time Convolution1. Solve the following for y(t.docx
Continuous Time Convolution1. Solve the following for y(t.docxmaxinesmith73660
 
Ray : modeling dynamic systems
Ray : modeling dynamic systemsRay : modeling dynamic systems
Ray : modeling dynamic systemsHouw Liong The
 
Determine if each system is1. memoryless2. Invertible3. Causal.pdf
Determine if each system is1. memoryless2. Invertible3. Causal.pdfDetermine if each system is1. memoryless2. Invertible3. Causal.pdf
Determine if each system is1. memoryless2. Invertible3. Causal.pdffashionfolionr
 
Eece 301 note set 14 fourier transform
Eece 301 note set 14 fourier transformEece 301 note set 14 fourier transform
Eece 301 note set 14 fourier transformSandilya Sridhara
 
EC202 SIGNALS & SYSTEMS Module1 QUESTION BANK
EC202 SIGNALS & SYSTEMS  Module1 QUESTION BANKEC202 SIGNALS & SYSTEMS  Module1 QUESTION BANK
EC202 SIGNALS & SYSTEMS Module1 QUESTION BANKVISHNUPRABHANKAIMAL
 

Similar to Contemporary Communication Systems Solutions Manual Chapter 2 (20)

Master
MasterMaster
Master
 
Master method
Master methodMaster method
Master method
 
Mathematical formula tables
Mathematical formula tablesMathematical formula tables
Mathematical formula tables
 
Operations on Continuous time Signals.
Operations on Continuous time Signals.Operations on Continuous time Signals.
Operations on Continuous time Signals.
 
233_Sample-Chapter.pdf
233_Sample-Chapter.pdf233_Sample-Chapter.pdf
233_Sample-Chapter.pdf
 
233_Sample-Chapter (1).pdf
233_Sample-Chapter (1).pdf233_Sample-Chapter (1).pdf
233_Sample-Chapter (1).pdf
 
Signal and Systems part i
Signal and Systems part iSignal and Systems part i
Signal and Systems part i
 
8fbf4451c622e6efbcf7452222d21ea5_MITRES_6_007S11_hw02.pdf
8fbf4451c622e6efbcf7452222d21ea5_MITRES_6_007S11_hw02.pdf8fbf4451c622e6efbcf7452222d21ea5_MITRES_6_007S11_hw02.pdf
8fbf4451c622e6efbcf7452222d21ea5_MITRES_6_007S11_hw02.pdf
 
University of manchester mathematical formula tables
University of manchester mathematical formula tablesUniversity of manchester mathematical formula tables
University of manchester mathematical formula tables
 
Stirling theorem
Stirling theoremStirling theorem
Stirling theorem
 
Master theorm practive problems with solutions
Master theorm practive problems with solutionsMaster theorm practive problems with solutions
Master theorm practive problems with solutions
 
Continuous Time Convolution1. Solve the following for y(t.docx
Continuous Time Convolution1. Solve the following for y(t.docxContinuous Time Convolution1. Solve the following for y(t.docx
Continuous Time Convolution1. Solve the following for y(t.docx
 
Ray : modeling dynamic systems
Ray : modeling dynamic systemsRay : modeling dynamic systems
Ray : modeling dynamic systems
 
002 ray modeling dynamic systems
002 ray modeling dynamic systems002 ray modeling dynamic systems
002 ray modeling dynamic systems
 
002 ray modeling dynamic systems
002 ray modeling dynamic systems002 ray modeling dynamic systems
002 ray modeling dynamic systems
 
501 lecture8
501 lecture8501 lecture8
501 lecture8
 
Determine if each system is1. memoryless2. Invertible3. Causal.pdf
Determine if each system is1. memoryless2. Invertible3. Causal.pdfDetermine if each system is1. memoryless2. Invertible3. Causal.pdf
Determine if each system is1. memoryless2. Invertible3. Causal.pdf
 
legendre.pptx
legendre.pptxlegendre.pptx
legendre.pptx
 
Eece 301 note set 14 fourier transform
Eece 301 note set 14 fourier transformEece 301 note set 14 fourier transform
Eece 301 note set 14 fourier transform
 
EC202 SIGNALS & SYSTEMS Module1 QUESTION BANK
EC202 SIGNALS & SYSTEMS  Module1 QUESTION BANKEC202 SIGNALS & SYSTEMS  Module1 QUESTION BANK
EC202 SIGNALS & SYSTEMS Module1 QUESTION BANK
 

Recently uploaded

Incoming and Outgoing Shipments in 1 STEP Using Odoo 17
Incoming and Outgoing Shipments in 1 STEP Using Odoo 17Incoming and Outgoing Shipments in 1 STEP Using Odoo 17
Incoming and Outgoing Shipments in 1 STEP Using Odoo 17Celine George
 
“Oh GOSH! Reflecting on Hackteria's Collaborative Practices in a Global Do-It...
“Oh GOSH! Reflecting on Hackteria's Collaborative Practices in a Global Do-It...“Oh GOSH! Reflecting on Hackteria's Collaborative Practices in a Global Do-It...
“Oh GOSH! Reflecting on Hackteria's Collaborative Practices in a Global Do-It...Marc Dusseiller Dusjagr
 
भारत-रोम व्यापार.pptx, Indo-Roman Trade,
भारत-रोम व्यापार.pptx, Indo-Roman Trade,भारत-रोम व्यापार.pptx, Indo-Roman Trade,
भारत-रोम व्यापार.pptx, Indo-Roman Trade,Virag Sontakke
 
Hierarchy of management that covers different levels of management
Hierarchy of management that covers different levels of managementHierarchy of management that covers different levels of management
Hierarchy of management that covers different levels of managementmkooblal
 
Employee wellbeing at the workplace.pptx
Employee wellbeing at the workplace.pptxEmployee wellbeing at the workplace.pptx
Employee wellbeing at the workplace.pptxNirmalaLoungPoorunde1
 
Full Stack Web Development Course for Beginners
Full Stack Web Development Course  for BeginnersFull Stack Web Development Course  for Beginners
Full Stack Web Development Course for BeginnersSabitha Banu
 
Final demo Grade 9 for demo Plan dessert.pptx
Final demo Grade 9 for demo Plan dessert.pptxFinal demo Grade 9 for demo Plan dessert.pptx
Final demo Grade 9 for demo Plan dessert.pptxAvyJaneVismanos
 
Types of Journalistic Writing Grade 8.pptx
Types of Journalistic Writing Grade 8.pptxTypes of Journalistic Writing Grade 8.pptx
Types of Journalistic Writing Grade 8.pptxEyham Joco
 
Framing an Appropriate Research Question 6b9b26d93da94caf993c038d9efcdedb.pdf
Framing an Appropriate Research Question 6b9b26d93da94caf993c038d9efcdedb.pdfFraming an Appropriate Research Question 6b9b26d93da94caf993c038d9efcdedb.pdf
Framing an Appropriate Research Question 6b9b26d93da94caf993c038d9efcdedb.pdfUjwalaBharambe
 
Roles & Responsibilities in Pharmacovigilance
Roles & Responsibilities in PharmacovigilanceRoles & Responsibilities in Pharmacovigilance
Roles & Responsibilities in PharmacovigilanceSamikshaHamane
 
History Class XII Ch. 3 Kinship, Caste and Class (1).pptx
History Class XII Ch. 3 Kinship, Caste and Class (1).pptxHistory Class XII Ch. 3 Kinship, Caste and Class (1).pptx
History Class XII Ch. 3 Kinship, Caste and Class (1).pptxsocialsciencegdgrohi
 
Historical philosophical, theoretical, and legal foundations of special and i...
Historical philosophical, theoretical, and legal foundations of special and i...Historical philosophical, theoretical, and legal foundations of special and i...
Historical philosophical, theoretical, and legal foundations of special and i...jaredbarbolino94
 
Organic Name Reactions for the students and aspirants of Chemistry12th.pptx
Organic Name Reactions  for the students and aspirants of Chemistry12th.pptxOrganic Name Reactions  for the students and aspirants of Chemistry12th.pptx
Organic Name Reactions for the students and aspirants of Chemistry12th.pptxVS Mahajan Coaching Centre
 
ECONOMIC CONTEXT - PAPER 1 Q3: NEWSPAPERS.pptx
ECONOMIC CONTEXT - PAPER 1 Q3: NEWSPAPERS.pptxECONOMIC CONTEXT - PAPER 1 Q3: NEWSPAPERS.pptx
ECONOMIC CONTEXT - PAPER 1 Q3: NEWSPAPERS.pptxiammrhaywood
 
How to Make a Pirate ship Primary Education.pptx
How to Make a Pirate ship Primary Education.pptxHow to Make a Pirate ship Primary Education.pptx
How to Make a Pirate ship Primary Education.pptxmanuelaromero2013
 
Crayon Activity Handout For the Crayon A
Crayon Activity Handout For the Crayon ACrayon Activity Handout For the Crayon A
Crayon Activity Handout For the Crayon AUnboundStockton
 
internship ppt on smartinternz platform as salesforce developer
internship ppt on smartinternz platform as salesforce developerinternship ppt on smartinternz platform as salesforce developer
internship ppt on smartinternz platform as salesforce developerunnathinaik
 
CARE OF CHILD IN INCUBATOR..........pptx
CARE OF CHILD IN INCUBATOR..........pptxCARE OF CHILD IN INCUBATOR..........pptx
CARE OF CHILD IN INCUBATOR..........pptxGaneshChakor2
 

Recently uploaded (20)

Incoming and Outgoing Shipments in 1 STEP Using Odoo 17
Incoming and Outgoing Shipments in 1 STEP Using Odoo 17Incoming and Outgoing Shipments in 1 STEP Using Odoo 17
Incoming and Outgoing Shipments in 1 STEP Using Odoo 17
 
“Oh GOSH! Reflecting on Hackteria's Collaborative Practices in a Global Do-It...
“Oh GOSH! Reflecting on Hackteria's Collaborative Practices in a Global Do-It...“Oh GOSH! Reflecting on Hackteria's Collaborative Practices in a Global Do-It...
“Oh GOSH! Reflecting on Hackteria's Collaborative Practices in a Global Do-It...
 
भारत-रोम व्यापार.pptx, Indo-Roman Trade,
भारत-रोम व्यापार.pptx, Indo-Roman Trade,भारत-रोम व्यापार.pptx, Indo-Roman Trade,
भारत-रोम व्यापार.pptx, Indo-Roman Trade,
 
Hierarchy of management that covers different levels of management
Hierarchy of management that covers different levels of managementHierarchy of management that covers different levels of management
Hierarchy of management that covers different levels of management
 
Model Call Girl in Tilak Nagar Delhi reach out to us at 🔝9953056974🔝
Model Call Girl in Tilak Nagar Delhi reach out to us at 🔝9953056974🔝Model Call Girl in Tilak Nagar Delhi reach out to us at 🔝9953056974🔝
Model Call Girl in Tilak Nagar Delhi reach out to us at 🔝9953056974🔝
 
Employee wellbeing at the workplace.pptx
Employee wellbeing at the workplace.pptxEmployee wellbeing at the workplace.pptx
Employee wellbeing at the workplace.pptx
 
Full Stack Web Development Course for Beginners
Full Stack Web Development Course  for BeginnersFull Stack Web Development Course  for Beginners
Full Stack Web Development Course for Beginners
 
Final demo Grade 9 for demo Plan dessert.pptx
Final demo Grade 9 for demo Plan dessert.pptxFinal demo Grade 9 for demo Plan dessert.pptx
Final demo Grade 9 for demo Plan dessert.pptx
 
Types of Journalistic Writing Grade 8.pptx
Types of Journalistic Writing Grade 8.pptxTypes of Journalistic Writing Grade 8.pptx
Types of Journalistic Writing Grade 8.pptx
 
Framing an Appropriate Research Question 6b9b26d93da94caf993c038d9efcdedb.pdf
Framing an Appropriate Research Question 6b9b26d93da94caf993c038d9efcdedb.pdfFraming an Appropriate Research Question 6b9b26d93da94caf993c038d9efcdedb.pdf
Framing an Appropriate Research Question 6b9b26d93da94caf993c038d9efcdedb.pdf
 
Roles & Responsibilities in Pharmacovigilance
Roles & Responsibilities in PharmacovigilanceRoles & Responsibilities in Pharmacovigilance
Roles & Responsibilities in Pharmacovigilance
 
History Class XII Ch. 3 Kinship, Caste and Class (1).pptx
History Class XII Ch. 3 Kinship, Caste and Class (1).pptxHistory Class XII Ch. 3 Kinship, Caste and Class (1).pptx
History Class XII Ch. 3 Kinship, Caste and Class (1).pptx
 
Historical philosophical, theoretical, and legal foundations of special and i...
Historical philosophical, theoretical, and legal foundations of special and i...Historical philosophical, theoretical, and legal foundations of special and i...
Historical philosophical, theoretical, and legal foundations of special and i...
 
Organic Name Reactions for the students and aspirants of Chemistry12th.pptx
Organic Name Reactions  for the students and aspirants of Chemistry12th.pptxOrganic Name Reactions  for the students and aspirants of Chemistry12th.pptx
Organic Name Reactions for the students and aspirants of Chemistry12th.pptx
 
ECONOMIC CONTEXT - PAPER 1 Q3: NEWSPAPERS.pptx
ECONOMIC CONTEXT - PAPER 1 Q3: NEWSPAPERS.pptxECONOMIC CONTEXT - PAPER 1 Q3: NEWSPAPERS.pptx
ECONOMIC CONTEXT - PAPER 1 Q3: NEWSPAPERS.pptx
 
How to Make a Pirate ship Primary Education.pptx
How to Make a Pirate ship Primary Education.pptxHow to Make a Pirate ship Primary Education.pptx
How to Make a Pirate ship Primary Education.pptx
 
Crayon Activity Handout For the Crayon A
Crayon Activity Handout For the Crayon ACrayon Activity Handout For the Crayon A
Crayon Activity Handout For the Crayon A
 
internship ppt on smartinternz platform as salesforce developer
internship ppt on smartinternz platform as salesforce developerinternship ppt on smartinternz platform as salesforce developer
internship ppt on smartinternz platform as salesforce developer
 
9953330565 Low Rate Call Girls In Rohini Delhi NCR
9953330565 Low Rate Call Girls In Rohini  Delhi NCR9953330565 Low Rate Call Girls In Rohini  Delhi NCR
9953330565 Low Rate Call Girls In Rohini Delhi NCR
 
CARE OF CHILD IN INCUBATOR..........pptx
CARE OF CHILD IN INCUBATOR..........pptxCARE OF CHILD IN INCUBATOR..........pptx
CARE OF CHILD IN INCUBATOR..........pptx
 

Contemporary Communication Systems Solutions Manual Chapter 2

  • 1. 2 4 Contemporary Communication Systems 1st Edition Mesiya Solutions Manual Download:http://testbanklive.com/download/contemporary-communication-systems- 1st-edition-mesiya-solutions-manual/ Chapter 2 2.1 Consider the signals di splayed in Fi gure P2.1.Show that each of these signals can be expressed as the sum of r ectangular Π (t ) and/or triangular Λ (t ) pulses. Figure P2.1 Solution: a. x (t ) ⎛ t ⎞ ⎛ t ⎞ 1 = Π ⎜ ⎟ + Π ⎜ ⎟ ⎝ ⎠ ⎝ ⎠ b. x (t ) = 2 Λ ⎛ t −3 ⎞ − Λ ⎛ t −3 ⎞ 2 ⎜ 6 ⎟ ⎜ 2 ⎟ ⎝ ⎠ ⎝ ⎠ c. x (t ) = ... + Π ⎛ t + 11 ⎞ + Π ⎛ t + 7 ⎞ + Π ⎛ t + 3 ⎞ + Π ⎛ t − 1 ⎞ + Π ⎛ t − 5 ⎞ + Π ⎛ t − 9 ⎞ + ...3 ⎜ 2 ⎟ ⎜ 2 ⎟ ⎜ 2 ⎟ ⎜ 2 ⎟ ⎜ 2 ⎟ ⎜ 2 ⎟ ⎝ ⎠ ⎝ ⎠ ⎝ ⎠ ⎝ ⎠ ⎝ ⎠ ⎝ ⎠ ∞
  • 2. = ∑ Π ⎡⎣ t − ( 2 n + 0.5 ) n =−∞ 1
  • 3. 2 2 2 2 2 x (t ) = 2 ⎢ 2 + 2 s gn ( t ) ⎥ 2 d. x (t ) = ... + Λ ⎛ t +4 ⎞ + Λ ⎛ t ⎞ + Λ ⎛ t −4 ⎞ + ...4 ⎜ 2 ⎟ ⎜ 2 ⎟ ⎜ 2 ⎟ ⎝ ⎠ ⎝ ⎠ ⎝ ⎠ ∞ ⎛ t −4 n ⎞ = ∑ Λ ⎜ ⎟ n =−∞ ⎝ 2 ⎠ 2.2 For the signal x (t ) in Figure P2.1 (b) plot the following signals a. x2 (t − 3) b. x2 ( −t ) c. x (2t ) d. x2 (3 − 2t ) Solution: x (t − 3) 1 x2 ( − t ) 1 0 1 2 3 4 5 6 7 8 9 −6 − 5 − 4 − 3 − 2 −1 0 t 1 2 3 x (2t ) 1 x (3 − 2t ) 1 0 t 1 2 3 4 5 6 − 2−1 0 t 1 2 3 4 2.3 Plot the following signals a. x1 (t ) = 2Π (t / 2) cos ( 6π t ) ⎡ 1 1 ⎤b. ⎣ ⎦
  • 4. 2
  • 5. t ) (t 4x t tt ) t( 1 x )t) (t(4 4x x tt 4 2 3 1 T T 1 T x1(t) x4( t) " 2 c. x3 (t ) = x2 ( −t + 2) d. x (t ) = s i n c ( 2t ) Π ( t / 2 ) Solution: 2 x (t ) 1. 5 21 0. 5 0 -0. 5 0 t 1 2 3 4 5 6 -1 -1. 5 -2 1 0. 8 -1 -0. 8 -0. 6 -0. 4 -0. 2 0 0. 2 0. 4 0. 6 0. 8 1 t 0. 6 x (t ) 2 " 0 t 1 2 3 4 0. 4 0. 2 0 -0. 2 -0. 4 -1 -0. 8 -0. 6 -0. 4 -0. 2 0 0. 2 0. 4 0. 6 0. 8 1 t 2.4 Determine whether the following signals are periodic. For periodic signals, determine the funda me ntal period. a. x1 (t ) = si n(π t ) + 5 cos( 4π t / 5) Solution: sin(π t ) is periodic with period T = 2π = 2 . cos( 4π t / 5) π is periodic with period T 2π 5 1 2 = 4π / 5 = 2 . x (t ) is peri odic if the ratio can be written as ratio of 2 integers. In the present case, T 2 × 2 41 = = 5 5
  • 6. 3
  • 7. 2 1 o 1 2 3 T 3 2 2 Therefore, x (t ) is periodic with fundame ntal period T such that 1 o T = 5T = 4T = 10 o 1 2 b. x (t ) = e j 3 t + e j 9 t + cos (12t ) Solution: e j 3 t is periodic with period T 2π j 9 t = 3 . Similarly, e is periodic with period T = 2π . cos(12t ) is periodic with period T = 2π = π . x (t ) is periodic with 2 9 3 2π 12 6 2 funda me ntal period T = L C M ( T , T , T ) = 3 . c. x3 (t ) = s i n( 2π t ) + cos(10t ) Solution: sin( 2π t ) is periodic with period T = 2π 1 2π = 1 . cos(10t ) is periodic with period T = 2π = π . x (t ) is peri odic if the ratio T1 can be written as ratio of integers. 2 10 5 3 In the present case, T 1 × 5 51 = = T2 π π Since π is an irrational number, the ratio is not rational. Therefore, periodic. d. x (t ) = cos ⎛ 2π t − π ⎞ + s i n(5π t ) x (t ) is not 4 ⎜ 4 ⎟ ⎝ ⎠ Solution: cos ⎛ 2π t − π ⎞ is periodic with period T 2π = = 1 . sin(5π t ) is periodic with ⎜ 4 ⎟ 1 2π⎝ ⎠ period T 2π
  • 8. 4 T = 5π 2 = 5 . x (t ) is peri odic if the ratio T1 can be written as ratio of 2 integers. In the present case, 4
  • 9. T2 T 1 × 5 51 = = 2 2 Therefore, x (t ) is periodic with funda me ntal period T such that 4 o T = 2T = 5T = 2 o 1 2 2.5 Classify the following signals as odd or even or neither. a. x (t ) = − 4t Solution: x ( − t ) = 4t = − ( −4t ) = − x (t ) . So x (t ) is odd. b. x (t ) = e − t Solution: e − t = e − − t . So x (t ) is even. c. x (t ) = 5 cos(3t ) Solution: Since cos(t ) is even, 5 cos(3t ) is also even. d. x (t ) = s i n(3t − π ) 2 Solution: x (t ) = si n(3t − π ) = − cos(3t ) which is even. 2 e. x (t ) = u (t ) Solution: u (t ) is neither even nor odd; For example, u (1) = 1 but u ( −1) = 0 ≠ − u (1) .
  • 10. 5
  • 11. 1 1 2 2 T 2 f. x (t ) = si n( 2t ) + cos( 2t ) Solution: x (t ) is neither even nor odd; For example, x (π / 8) = 2 but x ( −π / 8) = 0 ≠ − x (π / 8) . 2.6 Determine whether the following signals are energy or power, or neither and calculate the corresponding ener gy or power in the signal: a. x1 (t ) = u (t ) Solution: The normalized average power of a signal x (t ) is defined as 1 P = li m T /2 ∫ u (t ) 2 d t = li m 1 T / 2 1 T 1 ∫ 1 d t = li m = x1 T →∞ T T →∞ T− T /2 0 T →∞ T 2 2 Therefore, x (t ) is a power signal. b. x (t ) = 4 co s ( 2π t ) + 3 co s ( 4π t ) Solution: 4 cos ( 2π t ) is a power signal. 3 co s ( 4π t ) is also a power signal. Since x (t ) is sum of two power signals, it is a power signal. 1 Px = li m2 T →∞ T /2 ∫ − T /2 4 cos ( 2π t ) + 3 cos ( 4π t ) 2 dt = lim 1 T →∞ T T /2 ∫ − T /2 4 cos( 2π t ) + 3 cos ( 4π t ) dt 1 = li m T /2 ⎡16 cos 2 ( 2π t ) + 9 cos 2 ( 4π t ) + 24 cos( 2π t ) cos ( 4π t ) ⎤d t T →∞ T ∫ ⎣ ⎦ − T /2 Now li m 1 T /2 ∫ 8 16 co s 2 ( 2π t )dt = li m
  • 12. T /2 ∫ 8 T [1 + co s( 4 π t ) ]dt = li m = 8T →∞ T li m 1 T →∞ T − T /2 T /2 ∫ − T /2 T →∞ T 9 cos 2 ( 4π t )dt = 4.5 − T /2 T → ∞ T 6
  • 13. T ⎦ x ⎛ − ⎞ ⎝ ⎠ ⎛ − ⎞ ⎝ ⎠ 3 4 2 4 24 T /2 24 T /2 li m T →∞ T ∫− T /2 cos( 2π t ) cos ( 4π t )dt = lim T →∞ ∫− T /2 ⎡⎣ co s ( 6π t ) + co s ( 2π t ) ⎤dt = 0 Therefore, P = 8 + 4.5 = 12.52 1 c. x (t ) =3 t Solution: T /2 E = li m 1 / t 2 dt = li m T /2 ⎛ 1 t −2 dt = li m − T /2 ⎞ 2 = li m = 0 x3 T → ∞ ∫− T /2 T /2 T →∞ ∫− T /2 T / 2 T →∞ ⎜ ⎝ t ⎟− T /2 ⎠ T →∞ ⎜ T / 2 ⎟ T /2 1 P = li m 1 / t 2 dt = li m1 1 ⎛ 1 t −2 dt = li m − ⎞ 1 2 = li m = 0 x3 T →∞ T ∫− T /2 T →∞ T ∫− T / 2 T → ∞ T ⎜ ⎝ t ⎟− T /2 ⎠ T → ∞ T ⎜ T / 2 ⎟ x (t ) is neither an energy nor a power signal. d. x (t ) = e − α t u (t ) Solution: E = li m T /2 e − α t u (t ) dt = li m T /2 ⎛ e −2α t T /2 e −2α t dt = li m − ⎞ 1 = li m − e − α T + 1 1 = x T → ∞ ∫ T →∞ ∫ ⎜ T →∞ 2α ⎟ 2α T → ∞ ( ) 2α 4 ⎜ ⎟ − T /2 0 ⎝ 0 ⎠ Thus x (t ) is an energy signal. e. x5 (t ) = Π (t / 3) + Π (t ) Solution:
  • 14. 5 x (t ) 2 1 − 3/ 2 3 / 2 t 7
  • 15. 5 6 2 2 t j 10 t ∫ 4 t T /2 6 7 ∑ ⎣ T /2 −1/2 1/ 2 −1/ 2 E x = li m5 T →∞ ∫ − T /2 Π (t / 3) + Π (t ) 2 dt = ∫ 1d t + −3 / 2 ∫ −1/2 4 d t + ∫ 1d t − 3 /2 = 1 + 4 + 1 = 6 Thus x (t ) is an energy signal. f. x (t ) = 5e ( −2 t + j 10 π t ) u (t ) Solution: E x = li m6 T →∞ T /2 ∫ 5e ( −2 t + j 10 π t ) u (t ) dt = li m T → ∞ T /2 ∫ T / 2 2 5e − e π ) dt = li m 2 5 e − dt T →∞ − T /2 0 0 ⎛ = 25 li m ⎜ − e −4 t ⎞ ⎟ = 25 li m ( − e −2 T + 1) = 25 T →∞ ⎜ ⎝ 4 ⎟ 40 ⎠ T →∞ 4 Thus x (t ) is an energy signal. ∞ g. x (t ) = Λ ⎡ ( t − 4 n ) / 2 n =−∞ Solution: x (t ) is a periodic signal with period T = 4 .7 o To /2 1 1 1 P = ⎡ Λ 22 t / 2 ⎤ dt = 1 − t 2 dt = 1 1 − 2t + t 2 dt x7 T ∫ ⎣ ( ) ⎦ T ∫ ( ) 2 ∫ ( ) o − To /2 1 ⎛ 1 t 3 ⎞ o 0 0 1 1 1 = ⎜ t − t 2 + ⎟ = ⎛ 1 − 1 + ⎞ = 2 3 2 ⎜ 3 ⎟ 6 ⎝ ⎠ 0 ⎝ ⎠ 2.7 Evaluate the following expressions by us ing the properties of the delta function: a. x1 (t ) = δ (4t ) s i n(2t )
  • 16. Solution: 1 δ (4t ) = 4 δ (t ) 1 1 x (t ) = δ (t ) s i n ( 2t ) = δ (t ) s i n ( 0) = 01 4 4 8
  • 17. 4 4 b. x (t ) = δ (t ) cos ⎛ 30π t + π ⎞ 2 ⎜ 4 ⎟ ⎝ ⎠ Solution: x (t ) = δ (t ) cos ⎛ 30π t + π ⎞ = δ (t ) cos ⎛ 0 + π ⎞ = cos ⎛ π ⎞ δ (t )2 ⎜ 4 ⎟ ⎜ 4 ⎟ ⎜ 4 ⎟ ⎝ ⎠ ⎝ ⎠ ⎝ ⎠ c. x3 (t ) = δ (t )s i nc (t + 1) Solution: x3 (t ) = δ (t )s i nc (t + 1) = δ ( t )s i nc ( 0 + 1) = δ ( t )s i nc (1) = δ ( t ) × 0 = 0 d. x (t ) = δ (t − 2) e − t s i n ( 2.5π t ) Solution: x (t ) = δ (t − 2 ) e − t s i n ( 2 . 5π t ) = δ (t − 2) e −2 si n ( 2.5π × 2) = δ (t − 2) e −2 sin (π ) = 0 e. x5 (t ) = ∞ ∫ δ ( 2t )s i nc (t ) d t −∞ Solution: x (t ) δ ( 2t )si n c (t ) d t 1 δ (t )si n c (t ) d t ∞ ∞ 5 = ∫ = 2 ∫ −∞ −∞ 1 δ (t )sinc ( 0) d t 1 1 δ (t ) d t ∞ ∞= 2 ∫ = 2 ∫ = 2 −∞ −∞ f. x6 (t ) = ∞ ∫ δ (t − 3) cos (t ) d t −∞ Solution: x6 (t ) = ∞ ∞ ∫ δ (t − 3) cos (t ) dt = cos (3) ∫ −∞ − ∞
  • 18. δ (t − 3) dt = cos (3) 9
  • 19. 1 ⎛ 4 ⎞ 3 t − ×⎛ ⎞ τ g. x (t ) ∞ δ (2 t ) 1 dt 7 = ∫−∞ − 1 − t 3 Solution: ∞ 1 ∞ 1 x (t ) = ∫ δ ( 2 − t ) d t = ∫ δ (t − 2 ) d t7 1 t 3 1 t 3 −∞ − −∞ − δ (t 2) 1 d t 1 δ (t 2) d t 1 ∞ ∞= ∫ − 1 − 23 = − 7∫ − = − 7 −∞ −∞ h. x8 (t ) = ∞ ∫ δ (3t − 4) e −3 t d t −∞ Solution: ∞ ∞ x (t ) = ∫ δ (3t − 4 ) e −3 t dt = ∫ δ ⎜ t − ⎟ e − dt8 3 ⎝ 3 ⎠−∞ −∞ 1 ∞ = δ t − 4 ⎞ e 3 4 3 dt = e −4 ∞ δ ⎛ t − 4 dt = e −4 ∫ ⎜ ⎟ ∫ ⎜ ⎟ 3 ⎝ 3 ⎠ 3 ⎝ 3 ⎠ 3 −∞ −∞ i. x9 (t ) = δ '(t ) ⊗ Π (t ) Solution: ∞ x (t ) (t τ )δ '(τ )dτ ( 1) d (t τ ) 9 = ∫ Π − = − d Π − −∞ τ = 0 = Π ' (t ) = δ ( t + 0.5 ) − δ ( t − 0.5 ) 2.8 For each of the following continuous-time systems, determine whether or not the system is (1) linear, (2) time-invariant, (3) me moryless, and (4) casual. a. y (t ) = x (t − 1) Solution:
  • 20. The system is linear, time-invariant, causal, and has me mory. The system has me mory because current value of the output depends onthepreviousvalueof theinput. The system is causal because current value of the output does not depend on future inputs. To prove linearity, let response of the system to x (t ) is x (t ) = α x1 ( t ) + β x2 (t ) . The 10
  • 21. 1 1 2 2 y (t ) = x (t − 1) = α x1 (t − 1) + β x2 ( t − 1) = α y1 (t ) + β y 2 (t ) where x (t ) ⎯T ⎯→ y (t ) and x (t ) ⎯T ⎯→ y (t ) . To show that the syst em is time-invari ant, let input. The corresponding output is x1 (t ) = x (t − t o ) be the system y1 (t ) = x1 (t − 1) = x (t − 1 − t o ) = y (t − t o ) b. y (t ) = 3 x (t ) − 2 Solution: The system is nonlinear, time-invariant, causal, and me moryless. The system is me moryless because current value of the output depends only on the current value of the input. The system is causal because current value of the output does not depend on future inputs. To prove nonlinearity, let to x (t ) is x (t ) = α x1 ( t ) + β x2 (t ) . The response of the system y (t ) = 3 x (t ) − 2 = 3 [α x1 (t ) + β x2 (t ) ] − 2 ≠ α y1 (t ) + β y 2 (t ) = α [ 3 x1 (t ) − 2 ] + β [ 3 x2 ( t ) − 2 ] To show that the syst em is time-invari ant, let input. The corresponding output is x1 (t ) = x (t − t o ) be the system y1 (t ) = 3 x1 (t ) − 2 = 3 x (t − t o ) − 2 = y (t − t o ) c. y (t ) = x (t ) Solution: The system is nonlinear, time-invariant, causal, and me moryless. The system is me moryless because current value of the output depends only on the current value of the input. The system is causal because current value of the output does not depend on future inputs. To prove nonlinearity, let x (t ) is x (t ) = α x1 ( t ) + β x2 (t ) . The response of the system to
  • 22. 11
  • 23. 1 1 o o y (t ) = α x1 (t ) + β x2 (t ) ≤ α x1 (t ) + β x2 (t ) Because α x (t ) + β x (t ) = α x (t ) + β x (t ) ≠ α y (t ) + β y (t ) fo r all α an d β , 1 2 1 2 1 2 the system is nonlinear. To show that the syst em is time-invari ant, let input. The corresponding output is x1 (t ) = x (t − t o ) be the system y (t ) = x (t ) = x (t − t ) = y (t − t ) 1 1 o o d. y (t ) = [ co s ( 2t ) ] x (t ) Solution: The system is linear, time-variant, causal, and memoryless. The system is me moryless because current value of the output depends only on the current value of the input. The system is causal because current value of the output does not depend on future inputs. To prove nonlinearity, let to x (t ) is x (t ) = α x1 ( t ) + β x2 (t ) . The response of the system y (t ) = [ cos ( 2t ) ] x (t ) = [ cos ( 2t ) ] [α x1 (t ) + β x2 (t ) ] = α [ cos ( 2t ) ] x1 (t ) + β [ cos ( 2t ) ] x2 ( t ) = α y1 ( t ) + β y 2 (t ) To prove time-variance, let corresponding output is x1 (t ) = x (t − t o ) be the system input. The y (t ) = [ cos ( 2t ) ] x (t ) = [ cos ( 2t ) ] x ( t − t ) ≠ y ( t − t ) = cos 2 ( t − t o ) x ( t − t o ) e. y (t ) = e x ( t ) Solution: The system is nonlinear, time-invariant, causal, and me moryless. The system is me moryless because current value of the output de pends only on the current value of the input. The system is causal because current value of the output does not depend on future inputs.
  • 24. 12
  • 25. 1 2 1 1 1 o o o o o To prove nonlinearity, let to x (t ) is x (t ) = α x1 ( t ) + β x2 (t ) . The response of the system y (t ) = eα x1 ( t ) + β x2 ( t ) = eα x1 ( t ) e β x2 ( t ) ≠ α y (t ) + β y (t ) = α e x1 ( t ) + β e x2 ( t ) for all α and β To show that the syst em is time-invari ant, let input. The correspondi ng output is x1 (t ) = x (t − t o ) be the system y (t ) = e x1 ( t ) = e x ( t − to ) = y (t − t ) f. y (t ) = t x (t ) Solution: The system is linear, time-variant, causal, and me moryless. The system is me moryless because current value of the output de pends only on the current value of the input. The system is causal because current value of the output does not depend on future inputs. To prove nonlinearity, let x (t ) = α x1 ( t ) + β x2 (t ) . The response of the system to x (t ) is y (t ) = t [α x1 (t ) + β x2 ( t ) ] = α t x1 ( t ) + β t x2 (t ) = α y1 (t ) + β y 2 ( t ) To prove time-variance, let corresponding output is x1 (t ) = x (t − t o ) be the system input. The y (t ) = t x (t ) = t x ( t − t ) ≠ y (t − t ) = ( t − t ) x ( t − t ) t g. y (t ) = ∫ x ( 2τ ) dτ −∞ Solution: The system is linear, time-invariant, causal, and has me mory. The system has me mory because current value of the output depends only on the past values of the input. The system is causal because current value of the output does not depend on future inputs. To prove linearity, let response of the system to x (t ) is x (t ) = α x1 ( t ) + β x2 (t ) . The
  • 26. 13
  • 27. 1 ∫ 1 ∫ o t t t y (t ) = ∫ [α x1 ( 2τ ) + β x2 ( 2τ ) ] dτ = α ∫ x1 ( 2τ ) dτ + β ∫ x2 ( 2τ ) dτ = α y1 (t ) + β y 2 (t ) −∞ −∞ −∞ To check for time-variance, let corresponding output is x1 (t ) = x (t − t o ) be the system input. The t t y (t ) = x ( 2τ ) dτ = x [ 2(τ − t ) ] dτ = t − t o ∫ x ( 2α ) d α = y ( t − t o ) −∞ −∞ −∞ 2.9 Calculate the output y (t ) of the LTI system for the followi ng cases: a. x (t ) = e −2 t u (t ) and h (t ) = u (t − 2) − u (t − 4) Solution: h (t ) = u (t − 2) − u (t − 4) = Π ⎛ t −3 ⎞ ⎜ 2 ⎟ y (t ) h (τ ) x (t τ ) dτ ⎝ ⎠ e 2( t −τ ) u (t τ ) τ−3 dτ ∞ ∞ = − = − − Π ⎛ ⎞ ∫ ∫ ⎜ 2 ⎟ −∞ −∞ For t < 2 , there is no overlap and ⎝ ⎠ y (t ) = 0. t t − 2 e−2τ t − 2 1 e −2 ( t − 2 ) For 2 ≤ t < 4 , y (t ) = ∫ e −2( t −τ ) dτ = ∫ e −2τ dτ = = − −2 2 2 0 0 For t ≥ 4 , 4 4 4 e 2τ e 8 − e4 ( e 4 − 1) y (t ) = ∫ e −2( t −τ ) dτ = e −2 t ∫ e 2τ dτ =e −2 t = e −2 t = e −2 ( t − 2 ) 2 2 22 2 2
  • 28. ⎧1 − e −2( t − 2) ⎪ , 2 ≤ t ≤ 4 ⎪ 2 ⎪ ( e 4 − 1)y (t ) = ⎨ e −2( t − 2) , ⎪ 2 t > 4 ⎪ 0, otherwise ⎪ ⎪⎩ b. x (t ) = e − t u (t ) and h ( t ) = e −2 t u (t ) 14
  • 29. ⎨ ( ∫ Solution: ∞ t y (t ) = ∫ e −τ u (τ ) e −2( t −τ ) u (t − τ ) dτ = ∫ e −τ e − 2 ( t −τ ) dτ −∞ 0 For t < 0 , there is no overlap and y (t ) = 0. t − t τ − t τ t − t t − t − t y (t ) = e 2 ∫ e dτ =e 2 e 0 = e 2 e0 − 1) = e − e 2 , t ≥ 0 c. x (t ) = u ( − t ) an d h (t ) = δ ( t ) − 3e −2 t u (t ) Solution: y (t ) = ∞ ∞ h (τ ) x (t − τ )dτ = ⎡δ (τ ) − 3e −2τ u (τ ) ⎤ u (τ − t ) dτ ∫ ∫ ⎣ ⎦ −∞ −∞ Now ∞ ∫ δ (τ )u (τ − t ) dτ = u ( −t ) −∞ ∞ 3e −2τ ∞ 3 For t < 0 , 3e −2τ dτ = = −2 2 0 0 ∞ ∞ 3e −2τ ∞ 3 For t ≥ 0 , ∫ 3e −2τ u (τ )u (τ − t ) dτ = ∫ 3e −2τ dτ = = −2 2 e −2 t −∞ ⎧ 3 + 1 = 5 , y (t ) = ⎪ 2 2 3⎪ e −2 t , t t t < 0 t ≥ 0 2 d. x (t ) = δ ( t − 2) + 3e 3 t u ( −t ) and h (t ) = u (t ) − u (t − 1) Solution:
  • 30. 15
  • 31. 1 ⎜ e 1 ⎜ e ⎩ y (t ) = ∞ ∞ h (τ ) x (t − τ )dτ = ⎡δ (t − τ − 2 ) + 3e 3( t −τ ) u ( − t + τ ) ⎤ [ u (τ ) − u (τ − 1) ] dτ ∫ ∫ ⎣ ⎦ −∞ −∞ ∞ ∞ = ∫ δ (t − τ − 2) [ u (τ ) − u (τ − 1) ] dτ + ∫ 3e 3( t −τ ) u ( − t + τ ) [ u (τ ) − u (τ − 1) ] dτ −∞ −∞ Now ∞ ∫ δ (t − τ − 2) [ u (τ ) − u (τ − 1) ] dτ = u (t − 2) − u ( t − 2 − 1) = Π ( t − 2.5 ) −∞ For t < 0 , ∞ 1 1 ∫ 3e 3( t −τ ) u ( − t + τ ) [ u (τ ) − u (τ − 1) ] dτ = ∫ 3e 3( t −τ ) dτ = 3e 3 t ∫ e − 3τ dτ −∞ 0 0 For 0 < t ≤ 1 , = 3e 3 t e −3τ −3 0 = e 3 t ⎛ 1 − ⎝ 1 ⎞ 3 ⎟ ⎠ ∞ 1 1 ∫ 3e 3( t −τ ) u ( − t + τ ) [ u (τ ) − u (τ − 1) ] dτ = ∫ 3e 3( t −τ ) dτ = 3e 3 t ∫ e − 3τ dτ −∞ t t ⎧ e 3 t ⎛ 1 − 1 ⎞ , t ≤ 0 = 3e 3 t e −3τ −3 t = e 3 t ⎛ e −3 t − ⎝ 1 ⎞ 3 ⎟ ⎠ ⎪ ⎜ e 3 ⎟ ⎪ ⎝ ⎠ y (t ) = ⎪ e 3 t ⎛ e −3 t − 1 ⎞ , 0 < t ≤ 1⎨ ⎜ e 3 ⎟ ⎪ ⎝ ⎠ ⎪1, 2 ≤ t ≤ 3 ⎪ 0, ot herwi s e 2.10 The impulse response function of a continuous-time LTI is displayed in Fi gure P2.2(b). Assumi ng the input x (t ) to the system is waveform illustrated in Figure P2.2(a), determine the syst em output wavefor m y (t ) and sketch it. Solution:
  • 32. For t < 1 , there is no overlap and y (t ) = 0. 16
  • 33. Figure P2.2 x (τ ) (a) 1 0 1 2 3 τ 4 5 6 7 h (t − τ ) 1 (b) 1 < t < 2 τ 0 1 2 3 4 h (t − τ ) 1 (c) 5 < t < 6 τ 0 1 2 3 4 As shown in Figure (b), 1 t -1 y (t ) = ∫ dτ = t − 1 0 for 1 ≤ t < 2 For 2 ≤ t < 3 , y (t ) = ∫ dτ = 1 0 1 For 3 ≤ t < 4 , y (t ) = ∫ dτ =1 − t + 3 = 4 − t t − 3 Referring to Figure (c), t t y (t ) = ∫ dτ = 5 t − 5 for 5 ≤ t < 7 For 7 ≤ t < 8 , y (t ) = ∫ dτ = t − t + 2 = 2 t − 2 7 For 8 ≤ t < 1 0 , y ( t ) 2 y (t ) =
  • 34. 17 ∫ dτ = 7 − t + 3 = 10 − t t − 3 1 1 2 3 4 t 5 6 7 8 9 10
  • 35. C C 2.11 An LTI system has the impulse response h (t ) = e −0.5 ( t − 2 ) u (t − 2) . a. Is the system casual? Solution: Yes. The system is casual because h (t ) = 0 for t < 0 . b. Is the system stable? Solution: ∞ ∞ −0.5 x ∞ The system is stable because ∫ h (t ) dt = ∫ e e −0.5 x dx = = 2 −0.5 −∞ 0 0 c. Repeat parts (a) and (b) for h (t ) = e −0.5 ( t + 2 ) u (t + 2) . Solution: The system is not causal but stable. 2.12 Wr ite down the exponential Fourier series coe fficients of the signal x (t ) = 5 s in( 40π t ) + 7 cos ( 80π t − π / 2 ) − cos (160π t + π / 4 ) Solution: Applying the Euler’s formula, we get the following terms: ⎡ e j 40 π t − e − j 40 π t ⎤ ⎡ e j 80 π t e − jπ / 2 + e − j 80 π t e jπ / 2 ⎤ ⎡ e j 160 π t e jπ / 4 + e − j 160 π t e − jπ / 4 ⎤ x (t ) = 5 ⎢ ⎥ + 7 ⎢ ⎥ − ⎢ ⎥ ⎣ 2 j ⎦ ⎣ 2 ⎦ ⎣ 2 ⎦ = − j 2.5e j 40 π t + j 2.5e − j 40 π t − j 3.5e j 80 π t + j 3.5e − j 80π t − 0.5e jπ / 4 e j 160 π t − 0.5e − jπ / 4 e − j 160 π t The Fourier coefficients are 1 = − j 2.5, C −1 = j 2.5 2 = − j 3.5, C −2 = j 3.5
  • 36. C3 = − 0.5e jπ /4 , C−3 = −0.5e − jπ /4 a. Is x (t ) period ic? If so, what is its peri od? Solution: 18
  • 37. 19 Yes. ω = 40π ⇒ f = 20, T 1 1 = . The period is = 0.05 sec . o o o 20 20 2.13 A signal has the two-sided spectrum representation shown in Figure P2.3. Figure P2.3 a. Wr ite the equation for x(t ). Solution: x (t ) = 14 cos ⎛ 100π t − π ⎞ + 10 + 8 cos ⎛ 300π t − π ⎞ ⎜ 3 ⎟ ⎜ 2 ⎟ ⎝ ⎠ ⎝ ⎠ b. Is the signal periodic? If so, what is its period? Solution: Yes. It is periodic with fundame ntal period T = 1 = 0.02 .o 50 c. Does the signal have energy at DC? Solution: Yes as indicated by the presence of DC term C0 = 10 . 2.14 Wr ite down the complex exponential Fourier seri es for each of the periodic signals shown in Figure P2.4. Use odd or even symme t ry whenever possible. Solution: a. T = 3, f = 1 / 3 o o
  • 38. 0 3 ⎢ ∫ ∫ ⎥ 3 3 ⎣ ⎦ 1 0 ⎢ ∫ ⎥ 1 ⎡ ⎤ 1 ⎡ ⎤ ⎣ − j ⎜ ⎟ n , 1 ⎡ 2 3 ⎤ 1 C = 2d t − 1d t = ( 4 − 1) = 1 ⎣ 0 2 ⎦ 1 ⎡ 2 C = 2 πnt 2e − j 3 dt − 3 2 πnt e − j 3 dt ⎤ n ⎢ ∫ ∫ ⎥ ⎣ 0 2 ⎦ j 3 ⎡ 2 πnt 2 − j 2 πnt 3 ⎤ = ⎢ 2e 3 3 × 2π n ⎢⎣ − e 3 ⎥ ⎥⎦ 0 2 j = ⎡ 2e − j 4 π n / 3 − 2 − e − j 2 π n + e − j 4 π n / 3 ⎤ 2π n j 3 3e − j 2 π n / 3 ⎛ e + j 2 π n / 3 − e − j 2 π n / 3 ⎞ = ( e − j 4 π n / 3 − 1) = ⎜ ⎟ 2π n 3e −j2 πn/3 π n ⎝ 2 j ⎠ ⎛ 2πn ⎞ = sin , π n ⎝ 3 ⎠ n = ± 1, ±2, ..... b. T = 2, f = 1 / 2 o o 1 ⎡ 1 ⎤ 1 t 2 1 C = t dt = 2 2 2 = 4 (1 − 1) = 0 ⎣ −1 ⎦ −1 1 1 C = ∫ t e − jπ nt d t = − ∫ t d ( e − jπ n t )n 2 ⎢ ⎥ j 2π n ⎢ ⎥ ⎣ −1 ⎦ ⎣ −1 ⎦ 1 j ⎡ 1 = te − jπ n t − e − jπ n t d t ⎤ 2π n ⎢ −1 ∫ ⎥ − 1 ⎦ j ⎡ 1 1 ⎤ = e − jπ n + e + jπ n + e − jπ n t 2π n ⎢ jπ n −1 ⎥ ⎣ ⎦ j ⎡ e − jπ n e + jπ n e − jπ n e jπ n ⎤ = 2 ⎢ π n + π n + jπ 2 n 2 − jπ 2 n 2 ⎥ ⎣ ⎦ je − jπ n j ( − 1)= = n = ± 1, ± 2, ..... c. T π n = 2, f π n = 1 / 2 o o 2 A ⎡ 1 1 ⎤ ⎛ t 2 ⎞ ⎛ 1 ⎞ A
  • 39. 0C = 2 ⎢ ∫ (1 − t )dt ⎥ = A ⎜ t − 2⎟ = A ⎜ 1 − 2 ⎟ = 2 ⎣ 0 ⎦ ⎝ ⎠ 0 ⎝ ⎠ 20
  • 40. 3 n π n π n 1 ⎥⎦ = ⎤ ⎢ 4 ⎡ 1 ⎤ ⎡ ⎤ ⎢ 0 3 ⎥ Since x (t ) is an even function of time, A ⎡ 2 A 1 ⎤ 1 1 C = n = 2 2 ∫ (1 − t ) cos(π nt )dt ⎥ = A ∫ cos(π nt )dt − A ∫ t cos(π nt )dt ⎣ 0 ⎦ 0 0 1 1 A A ⎡ 1 ⎤ = 0 − ∫ td ( si n(π nt ) )dt = ⎢ − t si n(π nt ) 0 + ∫ sin(π nt )dt ⎥ 0 ⎣ 0 ⎦ A ⎡ co s (π nt ) ⎤ A A = ⎢ 0 − π n ⎢⎣ π n ⎥ = − π 2 n 2 [ cos(π n ) − 1] = π 2 n 2 [1 − cos(π n ) ] ⎧ 0, ⎪ ⎨ 2A , n even n odd π 2 n 2 d. T = 3, f = 1 / 3 o o 1 3 / 2 2 1 2A ⎡ C = t dt+ 1dt ⎤ = 2A ⎡ t + t 3/ 2 = 1 ⎡ 1 + 1 ⎤ = 2A 0 ⎢ ∫ ∫ ⎥ ⎣ 0 1 ⎦ 3 ⎢ 2⎣ 0 1 ⎥ 3 2 2 3⎦ Since x (t ) is an even function of time, A 2 A 1.5 2 A 1 3 / 2 C = n = ⎡ x (t ) cos( 2π nt / 3)dt ⎤ = ⎡ t cos( 2π nt / 3)dt + cos( 2π nt / 3)dt ⎤ n 2 ⎢ 3 ∫ 4 ⎥ 3 ⎢ ∫ ∫ ⎥ ⎣ 0 ⎦ ⎣ 0 1 ⎦ Now 1 1 1 3 ⎡ ⎤ 3 ⎡ 1 ⎤ ∫ t cos( 2π nt / 3)dt = 2π n ⎢ ∫ t d ( sin( 2π nt / 3) ) ⎥ = 2π n ⎢ t si n( 2π nt / 3) 0 − ∫ sin( 2π nt / 3)dt ⎥ 0 ⎣ 0 ⎦ ⎣ 0 ⎦ 3 3 = sin( 2π n / 3) + cos( 2π nt / 3) 2π n 2π n 0 3 3 = sin( 2π n / 3) + [ cos( 2π n / 3) − 1] 2π n 2π n
  • 41. 3/ 2 3 3/ 2 3 ∫ cos( 2π nt / 3)dt = sin( 2π nt / 3) = − si n( 2π n / 3) 1 2π n 1 2π n Substituting yields 21
  • 42. 2 5 T Tn ⎦ ⎦ C 2A ⎛ 3 ⎞ ⎡ ⎛ 2πn ⎞ ⎤ 3A ⎡ ⎛ 2πn ⎞ ⎤ C = ⎜ ⎟ ⎢ cos ⎜ ⎟ − 1 = ⎢ cos ⎜ ⎟ − 1 n 3 ⎝ 2π n ⎠ ⎣ ⎝ 3 ⎠ ⎥ 2π 2 n 2 ⎣ ⎝ 3 ⎠ ⎥ e. x (t ) = where ∞ ∑n =−∞ p ( t − 8 n ) p ( t ) = Π ⎣⎡ ( t − 2 ) / 2 + Π ⎣⎡ ( t − 2 ) / 4 − Π ⎡⎣ ( t − 6 ) / 2 − Π ⎡⎣ ( t − 6 ) / 4 The FT of the pulse shape p ( t ) over [ 0, To ] is given by To P ( f ) = ∫ p (t ) e − j 2 π ft d t 0 The FS coefficients of a periodic signal with basic pulse shape p ( t ) are gi ven by 1 To n = ∫o 0 p (t ) e − j 2 π nf o t d t Comparing yields C = 1 o Now P ( f ) f = n f o Π ⎡⎣ ( t − 2 ) / 2 Π ⎡⎣ ( t − 2 ) / 4 ⎤⎦Π ⎡⎣ ( t − 6 ) / 2 ⎤⎦Π ⎡⎣ ( t − 6 ) / 4 ⎤⎦ ←⎯ℑ → 2s i nc ( 2 f ) e − j 4 π f ←⎯ℑ → 4s i nc ( 4 f ) e − j 4 π f ←⎯ℑ → 2s i nc ( 2 f ) e − j 12 π f ←⎯ℑ → 4s i nc ( 4 f ) e − j 12 π f
  • 43. ⎣ ⎦ ⎣ ⎦ 5 Therefore, P ( f ) = 2si nc ( 2 f ) e − j 4 π f ⎡1 − e − j 8π f ⎤ + 4si nc ( 4 f ) e − j 4 π f ⎡1 − e − j 8π f ⎤ The FS coefficients of a periodic signal x (t ) are now obtained as 22
  • 44. T { ⎣ ⎦ ⎣ ⎦ 1 n T n x (ν )e − o o dν e j 2 π nf ν n C o T C o 1 n = 2si nc ( 2 nf o o ) e − j 4 π nf o ⎡1 − e − j 8π nf o ⎤ + 4si nc ( 4 nf ) e − j 4 π nf o ⎡1 − e − j 8 π nf o ⎤} = {sinc ( n / 4 ) e − jπ n /2 ⎡1 − e − jπ n ⎤ + 2sinc ( n / 2 ) e − jπ n /2 ⎡1 − e − jπ n ⎤}4 ⎣ ⎦ ⎣ ⎦ 2.15 For the rectangular pulse train in Figure 2.23, compute the Fourier coefficients of the new periodic signal y( t) given by a. y (t ) = x (t − 0.5To ) Solution: The FS expansion of a periodic pulse train ∞ x (t ) ⎡ ( t −nTo ) ⎤ of = ∑ Π ⎢ ⎥ rectangular pul ses is given by n =−∞ ⎣ τ ⎦ x (t ) = ∞ ∑n =−∞ C x e j 2 π nf o t where the exponential FS coefficients are C x = τ sinc ( nf τ ) n o o Let the FS expansion of a periodic pulse train y (t ) = x (t − 0.5To ) be expressed as y (t ) = where ∞ ∑n =−∞ C y e j 2 π nf o t 1 C y = y (t ) e − j 2 π n f o t dt = 1 x (t − 0.5T )e − j 2 π n f o t dt n T ∫ T ∫ o o To o To 1 j 2 π nf (ν + 0 . 5T ) − j 2 π n f o ( 0 .5 To ) 1 = ∫ = ∫ x (ν )e − o dν T To To = e − jπ n C x o x n
  • 45. Time Shifting introduces a linear phase shift in the FS coeffi cients; their magnitudes are not changed. 23
  • 46. j 2 π n f t ∫ x (t ) e o e o dt j 2 π tf ( n −1) = ⎞ n ∫ 2 b. y (t ) = x (t ) e j 2 π t / To Solution: 1 C y = 1 ∫ y (t ) e − o dt = j 2 π t f − j 2 π nf t n T T o To o To 1 = x (t )e − o dt To To C x n −1 c. y (t ) = x (α t ) Solution: 1 ( ) − π o 1 − π o C y = ∫ y t e j 2 n f t dt = ∫ x ( α t )e j 2 n f t dt n T T o To − j π n ⎛ f o ν o To 1 x ( )e ⎜ α ⎟ d = α T ∫ ν ⎝ ⎠ ν = C x o α To Time Scaling does not change FS coefficients but the FS itself has changed as the f 2 f 3 f harmonic components are now at the frequencies ± o , ± o , ± o , … α α α 2.16 Draw the one-sided power spectrum for the square wave in Figure P2.5 with duty cycle 50%. Figure P2.5 Solution:
  • 47. The FS expansion for the square wave is given by 24
  • 48. n T ( − jπ n − j 2 π n − jπ n ) = One-sidedPowerSpect rum C { C π n x (t ) = where ∞ ∑n =−∞ C e j 2π nf o t 1 n = T ∫ x (t ) e − j 2 π n f o t dt o To To /2 To 1 ⎧⎪ = T ⎨ ⎫⎪ A ∫ e − j 2 π nf o t dt − A ∫ e − j 2 π nf o t dt ⎬ o 0 To / 2 A e − j 2 π n f t To /2 e − j 2 π n f t o = T ( − j 2π nf ) o − o 0 To / 2 } o o jA = e − 1 − e + e 2π n Therefore, ⎧ − j2A , n odd ⎪ n ⎨ ⎩⎪ 0, ot herwise Averag e power in the frequency component at f = nf equals C 2 . Figure o n displays the one-sided power spectrum for the square wave. 0. 9 One-s i ded P ower S pec t rum of S quare wave 0. 8 0. 7 0. 6 0. 5 0. 4 0. 3 0. 2 0. 1 0 0 5 10 15 20 25 F requenc y (x f )o a. Calculate the normalized average power.
  • 50. n Total Power including current Fourier coefficient 1 0.8106 3 0.9006 5 0.9331 7 0.9496 9 0.9596 11 0.9663 13 0.9711 15 0.9747 17 0.9775 19 0.9798 21 0.9816 The norma lized average power for a periodic signal x (t ) is given by To /2 2 1 P = ∫ x (t ) 2 dt = ATo = A 2 x T To − To /2 o b. Determine the 98% power bandwidth of the pulse train. Solution: 98% Power bandwidt h = 21 × f o 2.17 Determine the Fourier transforms of the signals shown in Figure P2.6. Figure P2.6 26
  • 51. 1 1 ⎣ 1 ⎣ ⎦ 2 2 2 ⎣ 2 2 2 ⎣ Solution (a) The pulse x (t ) can be expressed as x (t ) = A ⎡ Π ( t + 0.5 ) − Π ( t − 0.5 ) Now Π ( t ) ←⎯ℑ → sinc ( f ) Applying the time-shifting property of the FT, we obtain Π ( t + 0.5 ) ←⎯ℑ → sinc ( f ) e jπ f Π ( t − 0.5 ) ←⎯ℑ → sinc ( f ) e − jπ f Adding jπ f − jπ f jπ f − jπ f X ( f ) = A ⎣⎡ s i nc ( f ) e − sinc ( f ) e = Asinc ( f ) ⎣⎡ e − e = Aj 2sinc ( f ) s in ( π f ) = j 2π f A ⎡ si nc 2 ( f ) ⎤ Solution (b) The pulse x (t ) can be expressed as x (t ) = Π ( 2t ) co s ( 2π t ) Now 1 Π (2t ) ←⎯ℑ → s i nc ( f / 2) 2 cos( 2π t ) ←⎯ℑ → 1 ⎡δ ( f − 1) + δ ( f + 1) X ( f ) = ℑ {Π (2t )} ⊗ ℑ {cos (2π t )} = 1 s i nc ( f / 2) ⊗ 1 ⎡δ ( f − 1) + δ ( f + 1) 1 = 4 ⎡ ⎣ si nc ⎡⎣ 0.5 ( f − 1)
  • 52. 3 + si nc ⎡⎣ 0.5 ( f + 1) ⎤ ⎦ Solution (c) The pulse x (t ) can be expressed as 27
  • 53. 3 − 1 2 ⎣ ⎦ 4 2 x (t ) = p ( t ) + p ( − t ) where p ( t ) = e − t ⎡⎣ u ( t ) − u ( t − 1) From Table 2.2, we have 1 e − t u (t ) ←⎯ℑ → 1 + j 2π f Using the time-shifting property of the FT, we obtain e − j 2 π f e − t u ( t − 1) = e −1 e − ( t −1) u ( t − 1) ←⎯ℑ → e − 1 = 1 + j 2π f e − (1+ j 2 π f ) 1 + j 2π f Combining 1 e − (1+ j 2 π f ) 1 p (t ) ←⎯ℑ → P ( f ) = − = ⎡1 − e − (1+ j 2 π f ) ⎤ 1 + j 2π f 1 + j 2π f 1 + j 2π f ⎣ ⎦ By time-reversal pr operty, p ( −t ) ←⎯ℑ → P ( − f ) = 1 1 − j 2π f ⎡1 e − (1− j 2 π f ) ⎤ ⎣ ⎦ X f = P f + P − f 1 = 1 − e 1 + 1 − e ( ) ( ) ( ) ⎡ − (1+ j 2 π f ) ⎤ ⎡ − (1− j 2 π f ) ⎤ 3 1 + j 2π f ⎣ ⎦ 1 − j 2π f ⎣ ⎦ = {(1 − j 2π f ) ⎡1 − e − (1+ j 2 π f ) ⎤ + (1 + j 2π f ) ⎡1 − e − (1− j 2 π f ) ⎤}1 + ( 2π f ) ⎣ ⎦ ⎣ ⎦ 2 = 1 + ( 2π f ) ⎡1 − e −1 cos ( 2π f ) + 2π f e −1 sin ( 2π f ) ⎤ Solution (d) The pulse x (t ) can be expressed as
  • 54. 4x (t ) = Π ( t / 2 ) + Π ( t / 4 ) Now 28
  • 55. 4 W c 2 ⎣ c c ⎣ ⎦ ⎣ ⎦ Π ( t / 2 ) ←⎯ℑ → 2s i nc ( 2 f ) Π ( t / 4 ) ←⎯ℑ → 4s i nc ( 4 f ) Adding X ( f ) = 2s i n c ( 2 f ) + 4s i n c ( 4 f ) 2.18 Use properties of the Fourier transform to compute the Four ier transform of following signals. a. sinc 2 (Wt ) Solution: Λ (t / τ ) ←⎯ℑ → τ s i nc 2 ⎛ fτ ⎞ 2 ⎜ 2 ⎟ ⎝ ⎠ Using the duality property, we obtain Wsinc 2 (Wt ) ←⎯ℑ →Λ ( f / 2W ) sinc 2 (Wt ) ←⎯ℑ → 1 Λ ( f / 2W ) Thus the Fourier transform of a sinc 2 pulse is a triangular function in frequency. b. Π (t / T ) cos( 2π f c t ) Solution: Π ( t / T ) ←⎯ℑ → T s i nc ( f T ) cos( 2π f t ) ←⎯ℑ → 1 ⎡δ ( f − f ) + δ ( f + f ) X ( f ) = ℑ Π t / T ⊗ ℑ co s ( 2π f t ) = T s i n c 1 f T ⊗ ⎡δ f − f + δ f + f ⎤ { ( )} { T c } ( ) 2 ⎣ ( c ) ( c ) ⎦ = 2 {sin c ⎡T ( f − f c ) ⎤ + si nc ⎡T ( f + f c ) ⎤} c. ( e − t cos 10π t ) u (t ) Solution:
  • 56. 29
  • 57. 2 2 ⎢ ( + π 2 ) + π − π 2 2 ⎥ − t 1 2 From Table 2.2, 1 e − t u (t ) ←⎯ℑ → 1 + j 2π f Now ℑ {( e − t u (t ) ) cos 10π t } = ℑ {( e − t u (t ) )} ⊗ ℑ {cos 10π t } 1 1 = 1 + j 2π f ⊗ 2 ⎡⎣δ ( f − 5 ) + δ ( f + 5 ) 1 ⎡ 1 1 ⎤ = ⎢ + ⎥ 2 ⎣ 1 + j 2π ( f − 5 ) 1 + j 2π ( f + 5 ) ⎦ 1 ⎡ 1 + j 2π ( f + 5 ) + 1 + j 2π ( f − 5 ) ⎤ = 2 ⎢ ⎢⎣ ⎥ (1 + j 2π f ) − ( j 2π 5 ) ⎥⎦ ⎡ 1 + j 2π f ⎤ = ⎢ 2 2 ⎥ (1 + j 2π f ) + 100π ⎡ 1 + j 2π f ⎤ d. te − t u (t ) = ⎢ ⎥ 1 100 j 4 f 4 f⎣ ⎦ Solution: Applying the differentiation in frequency domain property in (2.85), we obtain − t ℑ j d te u (t ) ←⎯→ 2π df ℑ {e u (t )} That is, j d ⎡ (1 + j 2π f ) − ⎤ j 1− t ⎣ ⎦ℑ {te u (t )} = 2π df 1 = 2 = − 2π j 2π (1 + j 2π f ) (1 + j 2π f ) e. e− π t 2
  • 59. e − π t e − j 2 π ft dt e − ( 2 ) dt 2 ∞ ∞ 2 2 2 ⎜ ⎟ 2 ⎣ ⎦ 2 X ( f ) ∞ ∞ 2 π t + j 2 ft = ∫ = ∫−∞ −∞ − π f 2 π f 2 Multiplying the ri ght hand side by e e yields X ( f ) e − πf e − π ( t 2 + j 2 f t − f 2 ) dt e − π f 2 e − π ( t + jf ) dt = ∫ = ∫−∞ −∞ Substituting t + j f = ν , we obtain X ( f ) = e − π f ∞ ∫ e − πν dν −∞ 1 f. 4si nc 2 (t ) cos (100π t ) Solution: 4si nc 2 ( t ) ←⎯ℑ → 4 Λ ⎛ f ⎞ ⎝ 2 ⎠ cos (100π t ) ←⎯ℑ → 1 ⎡δ ( f − 50 ) + δ ( f + 50 ) ⎤ X ( f ) = ℑ { 4sinc 2 ( t )} ⊗ ℑ {cos (100π t )} = 4 Λ ⎛ f ⎞ ⊗ 1 ⎡δ ( f − 50 ) + δ ( f + 50 ) ⎤ = 2 { Λ ⎡⎣ 0.5 ( f − 50 ) ⎜ ⎟ ⎝ ⎠ + Λ ⎡⎣ 0.5 ( f − 50 ) ⎤⎦} 2 ⎣ ⎦ 2.19 Find the following convolutions: a. sinc (Wt ) ⊗ si nc ( 2Wt ) Solution: We use the convolution property of Fourier transfor m in (2.79). x (t ) ⊗ y (t ) ←⎯ℑ → X ( f )Y ( f ) Now
  • 60. 1 sinc ( 2Wt ) ←⎯ℑ → Π ( f / 2W ) 2W 31
  • 61. ⎛ ⎞ 55 ⎝ ⎠ Therefore, 1 1 sinc (Wt ) ⊗ s i nc ( 2Wt ) ←⎯ℑ → Π ( f / W ) × Π ( f / 2W ) W 2W 1 = 2W 2 Π ( f / W ) b. sinc 2 (Wt ) ⊗ sinc ( 2Wt ) Solution: Again using the convolution property of Fourier transform sinc 2 (Wt ) ⊗ si nc ( 2Wt ) ←⎯ℑ → 1 Λ ( f / 2W ) × 1 Π ( f / 2W )W 2W 1 = 2W 2 Λ ( f / 2W ) 2.20 The FT of a signal 1 X ( f ) = 5 + j 2π f x (t ) is described by Determine the FT V ( f ) of the following signals: a. v (t ) = x (5t − 1) Solution: v (t ) = x (5t − 1) = x ⎡ 5 ⎛ t − 1 ⎞ ⎤ ⎢ ⎜ 5 ⎟ ⎥ ⎣ ⎝ ⎠ ⎦ Let 1 f 1 1 y (t ) = x (5t ) ←⎯ℑ → Y ( f ) = X =⎜ ⎟ 5 ( j 2π f / 5 ) + 5 1 = j 2π f + 25 v (t ) = y ⎛ t − 1 ⎞ ←⎯ℑ → V ( f ) = Y ( f ) e −j2 πf 5 = 1 −j2 πf e 5 ⎜ 5 ⎟ j 2π f + 2 5⎝ ⎠
  • 62. 32
  • 63. ⎡ + ⎤ ⎡ + ⎤ ⎟ b. v (t ) = x (t ) cos(100π t ) Solution: x (t ) e j 100 π t x (t ) e − j 100 π t v (t ) = x (t ) cos(100π t ) = ⎣ ⎦ 2 Now x (t ) e j 100 π t x (t ) e − j 100 π t ⎣ ⎦ 2 1 ←⎯ℑ → V ( f ) = 2 X ( f − 50 ) + X ( f + 50 ) ⎤⎦ 1 1 = j 2π ( f − 50 ) + 5 + j 2π ( f + 50 ) + 5 After simplification, we get j2πf +5 V ( f ) = j 20π f + ( π 2 1 0 4 + 25 − 4π 2 f 2 ) c. v (t ) = x (t ) e j 10 t Solution: v (t ) = x (t ) e j 10 t ←⎯ℑ → V ( f ) = X ⎛ f − 5 ⎞ = 1 ⎜ ⎟ ⎛ ⎞⎝ π ⎠ j 2π ⎜ 5 f − + 5 π⎝ ⎠ dx (t) d. v (t ) = dt Solution: Using the differentiation property of FT d x (t ) ←⎯ℑ → j 2πfX ( f ) , we obtain dt V ( f ) = j2πf j 2π f X ( f ) = 5 + j 2π f e . v (t ) = x (t ) ⊗ u (t ) Solution: Using the convolution property of FT x (t ) ⊗ y (t ) ←⎯ℑ → X ( f )Y ( f ) , we can write
  • 64. 33
  • 65. 1 ⎛ 1 1 ⎞ V ( f ) = X ( f )U ( f ) = δ ( f ) + 5 + j 2π f ⎜ 2 j 2π f ⎟ ⎝ ⎠ 1 1 1 = 2 δ ( f ) 5 + j 2π f + j 2π f ( 5 + j 2π f ) 1 1 = δ ( f ) + 10 j 2π f ( 5 + j 2π f ) 2.21 Consider the delay eleme nt y (t ) = x (t − 3) . a. What is the impulse response h( t)? Solution: Si nc e y (t ) = h (t ) ⊗ x (t ) = x (t − 3) , the impulse response of the delay eleme nt is given from ( 2.16) as h (t ) = δ (t − 3) b. What is the ma gnitude and phase response function of the system? Solution: H ( f ) = ℑ {δ (t − 3)} = e − j 6 π f H ( f ) = 1 (H ( f ) = − 6π f 2.22 The periodic input x (t ) to an LTI system is displaye d in Figure P2.7. The frequency response function of the system is given by 2 H ( f ) = 2 + j 2π f Figure P2.7
  • 66. 34
  • 67. o T o o T To Cn o a. Wr ite the complex exponential FS of input x (t ) . Solution: The input x (t ) is rectangular pulse train in Example 2.24 shifted by T / 4 and duty cycle τ = 0.5 . That is, o x (t ) g ( t T ∞ / 4 ) ⎡ ( t −nTo − To /4) ⎤ = To − o = ∑ Π ⎢ 0.5T ⎥ n =−∞ ⎣ o ⎦ The comple x exponential FS of τ g (t ) from Exa mple 2.24 withTo T = 2 ( f = 0.5 ) and = 0.5 is given by o ∞ g (t ) = 0.5 ∑ s i nc ( 0.5 n )e jπ nt n =−∞ In Exercise 2.15(a), we showed that time shifting introduces a linear phase shift in the FS coefficients; their ma gnitudes are not changed. The phase shift is equal to e − j 2 π nf o u for a time shift of u. The exponential FS coefficients x (t ) are = 0.5s i nc ( 0.5n ) e − j 2 π nf o ( 0. 2 5To ) = e − jπ n /2 0.5s inc ( 0.5n ) ∞ x (t ) = 0.5 ⎡ e − jπ n /2 s i nc ( 0.5 n ) ⎤e jπ nt ∑ ⎣ ⎦ n =−∞ b. Plot the ma gnitude and phase response functions for H ( f ) . Solution: H ( f ) = 2 = 1 2 + j 2π f 1 + jπ f 1 1 H ( nf o ) = 1 + jπ f = 1 + jπ nf
  • 68. 35
  • 69. 0 2 4 6 8 0 2 4 6 8 F r equenc y (f) F r eq uenc y ( f) n =−∞ n = −∞ ⎢ ⎥ ∑ ⎢ ⎥ MagnitudeRespons e(dB) PhaseRespons e(degrees) 0 0 -1 0 -5 -2 0 -1 0 -3 0 -1 5 -4 0 -5 0 -2 0 -6 0 -2 5 -7 0 -8 0 -3 0 -9 0 c. Compute the complex exponential FS of the output y (t ) . Solution: Using (2.114), the output of an LTI system to the input x (t ) = ∞ ⎡ 0.5e − jπ n /2 s i nc ( 0.5 n ) ⎤e jπ nt ∑ ⎦ Cn is given by ⎡ ⎤ ∞ ∞ ⎢ 0.5e − jπ n /2 ⎥ y (t ) = ∑ C n H ( 0.5n ) e n =−∞ jπ n t = s inc ( 0.5n ) e jπ n t 1 + j 05π n FS co effi cien t o f y ( t ) 2.23 The frequency response of an ideal LP filter is given by ⎧⎪5e − j 0.0025π f , H ( f ) = ⎨ f < 1000 Hz 0, f > 1000 Hz Determine the output signal in each of the followi ng cases:
  • 70. a. x (t ) = 5 sin ( 400π t ) + 2 cos (1200π t − π / 2 ) − cos ( 2200π t + π / 4 ) 36
  • 71. ⎝ ⎠ ⎜ 2200 ⎟ ⎜ 2000 ⎟ ⎜ 2000 ⎟ Solution: 5, f < 1000 Hz H ( f ) = ⎨ ⎪⎩ ⎧⎪ 0, −0.0025π f , f > 1000 Hz f < 1000 Hz (H ( f ) = ⎨ ⎪⎩ 0, f > 1000 Hz Since H ( 200) = 5 and (H ( 200) = −π / 2 , the output of the system for an input 5sin(400πt ) can now be expressed using (2.117) as 25sin(400πt − π/2). Next H ( 600) = 5 and (H ( 600) = −3π / 2 = π / 2 , the output of the system for an input 2 cos (1200π t − π / 2 ) can now be expressed using (2.117) as 10 cos (1200π t − π / 2 + π / 2 ) = 10 cos (1200π t ) . Now H (1100) = 0. So the LP filter doesn’t pass cos ( 2200π t + π / 4 ) . The output of the LP filter is therefore given by y (t ) = 25 sin ( 400π t − π / 2 ) + 10 cos (1200π t ) sin( 2200π t )b. x (t ) = 2 si n( 400π t ) + π t Solution: Since H ( 200) = 5 and (H ( 200) = −π / 2 , the output of the system for an input 2sin(400πt ) is 10sin(400πt − π/2). To calculate the response to sin( 2200π t) , we note that π t sin( 2200π t) ⎛ f ⎞= 2200 × sinc ( 2200t ) ←⎯ℑ →Π π t ⎜ 2200 ⎟ In fr equency domain, the output of LP filter to sin( 2200π t) is π t Π ⎛ f ⎞ 5e − j 0.0025 π f Π ⎛ f ⎞ = 5Π ⎛ f ⎞ e − j 0.0025π f ⎝ ⎠ ⎝ ⎠ ⎝ ⎠
  • 72. 37
  • 73. ⎜ ⎟ ⎝ ⎠ ⎜ 1000 ⎟ ⎜ 1000 ⎟ ⎜ ⎟ ⎣ ⎦ Now Π ⎛ f ⎞ e − j 0.0025 π f ←⎯ℑ → 2000sinc ⎝ 2000 ⎠ 2000 ( t − 0.00125 ) The output of the LP filter is therefore given by y (t ) = 10 sin ( 400π t − π / 2 ) + 10000si nc sin(1000π t)c. x (t ) = cos( 400π t ) + π t 2000 ( t − 0.00125 ) Solution: Since H ( 200) = 5 and (H ( 200) = −π / 2 , the output of the system for an input cos( 400π t ) is 5 cos ( 400π t − π / 2 ) . To calculate the response to sin(1000πt) , we note that π t sin(1000π t) ⎛ f ⎞= 1000 × si nc (1000t ) ←⎯ℑ →Π π t ⎜ 1000 ⎟ In fr equency domain, the output of LP filter to 5e − j 0.0025π f Π ⎛ f ⎞ = 5Π ⎛ f ⎞ e − j 0.0025π f ⎝ ⎠ ⎝ ⎠ Now sin(1000πt) is π t Π ⎛ f ⎞ e − j 0.0025 π f ←⎯ℑ → 1000sinc ⎡1000 ( t − 0.00125 ) ⎤ ⎝ 1000 ⎠ ⎣ ⎦ The output of the LP filter is therefore given by y (t ) = 5 cos ( 400π t − π / 2 ) + 5000si nc ⎡1000 ( t − 0.00125 ) ⎤ d. x (t ) = 5 cos(800π t ) + 2δ (t ) Solution:
  • 74. 38
  • 75. ⎨ 4, Since H ( 400) = 5 and (H ( 4 00) = −π , the output of the system for an input 5 cos ( 800π t ) is 25 cos ( 800π t − π ) . The response of the system to δ (t ) is h (t ) . The impulse response of the ideal LP filter is 10000sinc Co mbining 2000 ( t − 0.00125 ) ⎤⎦ . y (t ) = 25 cos ( 800π t − π ) + 2 h (t ) = 25 cos ( 800π t − π ) + 20 × 103 sinc 2000 ( t − 0.00125 ) 2.24 The frequency response of an ideal HP filter is given by H ( f ) = ⎧⎪ ⎪⎩ 0, f > 20 Hz , f < 20 Hz Determine the output signal y (t ) for the input a. x (t ) = 5 + 2 cos ( 5 0π t − π / 2 ) − cos ( 75π t + π / 4 ) Solution: y (t ) = 8 cos ( 50π t − π / 2 ) − 4 cos ( 75π t + π / 4 ) b. x (t ) = cos ( 2 0π t − 3π / 4 ) + 3 cos (100π t + π / 4 ) Solution: y (t ) = 12 cos (100π t + π / 4 ) 2.25 The frequency response of an ideal BP filter is gi ven by H ( f ) = ⎧⎪ ⎨ ⎪⎩ 2e − j 0.0005 π f , 900< f 0, otherwise < 1000 Hz , Determine the output signal y (t ) for the input
  • 76. a. x (t ) = 2 cos (1850π t − π / 2 ) − cos (1900π t + π / 4 ) Solution: H (925) = 2 and H (950) = 2 . 39
  • 77. ⎝ ⎠ ⎡ ⎛ − ⎞ ⎛ + ⎞ ⎤ ⎛ − ⎞ ⎛ − ⎞ ⎛ + ⎞ (H ( f ) = − 0.0005π f . Therefore, (H (925) = − 0.0005π f (H (950) = −0.0005π f = −0.0005π × 925 = − 0.462π = − 0.0005π × 950 = − 0.475π y (t ) = 4 cos (1850π t − π / 2 − 0.462π ) − 2 cos (1900π t + π / 4 − 0.475π ) b. x (t ) = si nc ( 60t ) cos (1900π t ) Solution: 1 ⎛ f ⎞ 1 X ( f ) = 60 Π ⎜ 60 ⎟ ⊗ 2 [δ ( f − 950) + δ ( f + 950) ] 1 ⎡ ⎛ f −950 ⎞ ⎛ f +950 ⎞ ⎤ = Π + Π 120 ⎢ ⎜ 60 ⎟ ⎜ 60 ⎟ ⎥ ⎣ ⎝ ⎠ ⎝ ⎠ ⎦ Y ( f ) = H ( f ) X ( f ) 1 f 950 f 950 = 2 e − j 0. 0005 π f × × Π + Π 120 ⎢ ⎜ 60 ⎟ ⎜ 60 ⎟ ⎥ ⎣ ⎝ ⎠ ⎝ ⎠ ⎦ Now 1 f 950 ×Π ←⎯→ s i nc 60t e ⎜ ⎟ ℑ ( ) j 1900 π t 60 ⎝ 60 ⎠ 1 f 950 e ×Π ←⎯→ si nc ⎡ 60 t − 0.00025 ⎤ e − j 0.0005π f ℑ ⎜ ⎟ ( ) j 1900 π ( t − 0.00025 ) 60 ⎝ 60 ⎠ ⎣ ⎦ Similarly 1 f 950 ×Π ←⎯→ s i nc 60t e ⎜ ⎟ ℑ ( ) − j 1900 π t 60 ⎝ 60 ⎠
  • 78. ⎛ + ⎞ ⎣ ⎦ ⎣ ⎦ ⎣ 1 f 950 e ×Π ←⎯→ si nc ⎡ 60 t − 0.00025 ⎤ e − j 0.0005π f ℑ ⎜ ⎟ ( ) − j 1900 π ( t − 0.00025 ) 60 ⎝ 60 ⎠ ⎣ ⎦ Therefore, y (t ) = sinc ⎡ 60 ( t − 0.00025 ) ⎤ ⎡ e j 1900 π ( t − 0.00025 ) + e − j 1900 π ( t − 0.00025 ) ⎤ = 2s inc ⎡⎣ 60 ( t − 0.00025 ) cos ⎡1900π ( t − 0.00025 ) 40
  • 79. ⎝ ⎠ ⎡ ⎛ − ⎞ ⎛ + ⎞ ⎤ ⎛ + ⎞ ⎛ + ⎞ ⎣ ⎦ ⎣ ⎦ c. x (t ) = si nc 2 ( 30t ) cos (1900π t ) Solution: 1 ⎛ f ⎞ 1 X ( f ) = 30 Λ ⎜ 60 ⎟ ⊗ 2 [δ ( f − 950) + δ ( f + 950) ] 1 ⎡ ⎛ f −950 ⎞ ⎛ f +950 ⎞ ⎤ = Λ + Λ 60 ⎢ ⎜ 60 ⎟ ⎜ 60 ⎟ ⎥ ⎣ ⎝ ⎠ ⎝ ⎠ ⎦ Y ( f ) = H ( f ) X ( f ) 1 f 950 f 950 = 2 e − j 0. 0005π f × Λ + Λ 60 ⎢ ⎜ 60 ⎟ ⎜ 60 ⎟ ⎥ ⎣ ⎝ ⎠ ⎝ ⎠ ⎦ Now 1 ⎛ f −950 ⎞ ←⎯→ sinc 30t e ×Λ ⎜ ⎟ ℑ 2 ( ) j 1900 π t 30 ⎝ e 60 1 ⎠ ⎛ f −950 ⎞ ←⎯→ sinc ⎡ 30 t − 0.00025 ⎤ e − j 0.0005π f ×Λ ⎜ ⎟ ℑ 2 ( ) j 1900 π ( t − 0.00025 ) 30 ⎝ 60 ⎠ ⎣ ⎦ Similarly 1 f 950 ×Π ←⎯→ s i nc 30t e ⎜ ⎟ ℑ 2 ( ) − j 1 90 0 π t 30 ⎝ 60 ⎠ 1 f 950 e ×Π ←⎯→ si nc ⎡ 30 t − 0.00025 ⎤ e − j 0.0005π f ⎜ ⎟ ℑ 2 ( ) − j 1900 π ( t − 0.00025 ) 30 ⎝ 60 ⎠ ⎣ ⎦ Therefore, y (t ) = sinc 2 ⎡ 30 ( t − 0.00025 ) ⎤ ⎡ e j 1900 π ( t − 0. 00025 ) + e − j 1900 π ( t − 0. 00025 ) ⎤
  • 80. ⎣= 2s inc 2 ⎡⎣ 30 ( t − 0.00025 ) cos ⎡1900π ( t − 0.00025 ) 2.26 The signal 2e −2 t u (t ) is input to an ideal LP filter with passband edge frequency equal to 5 Hz . Find the energy density spectrum of the output of the filter. Calculat e the energy of the input signal and the output signal. Solution: 41
  • 81. ⎜ 10 ⎟ 10 y 2 2 ⎝ ⎠ −1 π 0 ∞ E = 2e −2 t u (t ) 2 dt = ∞ 4e −4 t dt =4 e −4 t ∞ = 1 x ∫ ∫ −4−∞ 0 0 2 x (t ) = 2e −2 t u (t ) ←⎯ℑ → 2 + j 2π f The energy density spectrum of the output y( t ), Y ( f ) 2 , is related to the energy density spectrum of the input x (t ) Y ( f ) 2 = H ( f ) 2 X ( f ) 2 = Π ⎛ f ⎞ ⎝ ⎠ 2 2 + j 2π f ∞ ∞ ∞ 2 2 E = y (t ) 2 dt = Y ( f ) 2 df = Π ⎛ f ⎞ 2 df y ∫ ∫ ∫−∞ −∞ −∞ ⎜ ⎟ 2 + j 2π f 5 1 = ∫ 1 + jπ f 2 5 df = 2 ∫ 1 df 1 + π 2 f 2 −5 0 Making change of variables π f = u ⇒ d f 1 = d u , we obtain π 2 5π 1 2 5π2 E = ∫0 du = tan 1 + u 2 π ( u ) = (1.507 ) = 0.9594 π Thus output of the LP filter cont ains 96% of the input signal energy. 2.27 Calculate and sketch the power spectral density of the following signals. Calculate the normalized average power of the signal in each case. a. x (t ) = 2 cos (1000π t − π / 2 ) − cos (1850π t + π / 4 ) Solution: x1 ( t ) x2 ( t ) 1 R (τ ) = li m T /2 x (t ) x (t − τ ) d t= R (τ ) +R (τ ) −R (τ ) −R (τ ) x T →∞ T ∫− T /2 x1 x2 x1 x2 x2 x1
  • 83. ⎣ ⎦ 2 2 2 T 2 1 R (τ ) = li m T /2 x (t ) x (t − τ ) dt x1 T →∞ T 1 ∫ 1 1 − T /2 T /2 ⎛ π ⎞ ⎡ π ⎤ = li m T ∫ 2 cos ⎜ 1000π t − 2 ⎟ 2 cos 1000π ( t − τ ) − 2 d t T →∞ ⎢ ⎥ − T /2 ⎝ ⎠ ⎣ ⎦ T /2 1 = 2 lim T →∞ T ∫ − T /2 {cos (1000πτ ) + cos ⎡1000π ( 2t − τ ) − π ⎤} d t = 2 cos (1000πτ ) The second term is zero because it integrates a sinusoidal func tion over a period. Similarly, 1 Rx (τ ) = li m T T /2 ∫x (t ) x (t − τ ) dt = 1 cos (1850πτ ) 2 T →∞ − T /2 The cross-correlation term 1 Rx x (τ ) = li m T T /2 x (t ) x (t − τ ) = lim 1 1 2 T T /2 2 cos (1000π t − π / 2 ) cos ⎡1850π ( t − τ ) + π/ 4 ⎤dt 1 2 T →∞ ∫ − T /2 1 T /2 T →∞ ∫ ⎣ ⎦ − T /2 = li m T →∞ ∫ − T /2 {cos ( 850π t − 1850πτ + 3π / 4 ) + s in ( 2850π t − 1850πτ − π / 4 )}dt is zero because it integrates a si nusoi dal function over a period in each case. Similarly, it can be shown that all ot he r cross-correlation terms are zero. Therefore, 1 Rx (τ ) = Rx1 (τ ) + Rx2 (τ ) = 2 cos (1000πτ ) + 2 cos (1850πτ ) ⎧ 1 ⎫Gx ( f ) = ℑ { Rx (τ )} = ℑ ⎨ 2 cos (1000πτ ) + cos (1850πτ ) ⎬ ⎩ ⎭ 1 = [δ ( f − 500) + δ ( f + 500) ] + 4 [δ ( f − 925) + δ ( f + 925) ] The norma lized average power is obtained by using (2.172) as
  • 84. 43
  • 85. x ∫ x PowerSpectralDens ity ∞ ∞ 1 ∞ P = G ( f )df = ∫ [δ ( f − 500) + δ ( f + 500) ] df + 4 ∫ [δ ( f − 925) + δ ( f + 925) ] df −∞ −∞ −∞ 1 1 5 = 1 + 1 + 4 + 4 = 2 1 0. 8 0. 6 0. 4 0. 2 0 -1 -0. 8 -0. 6 -0. 4 -0. 2 0 0. 2 0. 4 0. 6 0. 8 1 F requenc y (k Hz ) b. x (t ) = [1 + sin( 200π t ) ] cos( 2000π t ) Solution: x (t ) = [1 + s in( 200π t ) ] cos( 2000π t ) = cos( 2000π t ) + s in( 200π t ) cos( 2000π t ) 1 1 = cos( 2000π t ) + 2 sin ( 2200π t ) − 2 sin (1800π t ) x1 ( t ) Now x2 ( t ) x3 ( t ) 1 R (τ ) = li m T /2 x (t ) x (t − τ ) d t= R (τ ) +R (τ ) +R (τ ) x T →∞ T ∫− T /2 x1 x2 x3 where 1 Rx1 (τ ) = 2 cos ( 2000πτ ) 1 Rx2 (τ ) = 8 cos ( 2200πτ ) 1
  • 86. Rx3 (τ ) = 8 cos (1800πτ ) 44
  • 87. ⎩ ⎭ x ∫ x PowerSpectralDens ity Therefore, 1 1 1 Rx (τ ) = 2 cos ( 2000πτ ) + 8 cos ( 2200πτ ) + 8 cos (1800πτ ) ⎧ 1 1 1 ⎫ Gx ( f ) = ℑ { Rx (τ )} = ℑ ⎨ 2 cos ( 2000πτ ) + 8 cos ( 2200πτ ) + 8 cos (1800πτ ) ⎬ 1 1 = 4 [δ ( f − 1000) + δ ( f + 1000) ] + 16 [δ ( f − 1100) + δ ( f + 1100) ] 1 + 16 [δ ( f − 900) + δ ( f + 900) ] The normalized average power is obtained by using (2.172) as ∞ 1 ∞ 1 ∞ P = G ( f )df = 4 ∫ [δ ( f − 1000) + δ ( f + 1000) ] df + 16 ∫ [δ ( f − 1100) + δ ( f + 1100) ] df −∞ −∞ −∞ 1 ∞ 1 1 1 3 + 16 ∫ [δ ( f − 900) + δ ( f + 900) ] d f −∞ = 2 + 8 + 8 = 4 0. 25 0. 2 0. 15 0. 1 0. 05 0 -1 -0. 8 -0. 6 -0. 4 -0. 2 0 0. 2 0. 4 0. 6 0. 8 1 Frequenc y (k Hz ) c. x (t ) = cos 2 ( 200π t ) s i n (1800π t ) Solution:
  • 88. 45
  • 89. 2 ⎣ ⎦ 2 2 1 2 3 ⎩ ⎭ x ∫ x x (t ) = 1 ⎡1 + cos ( 400π t ) ⎤ sin (1800π t ) = 1 si n (1800π t ) + 1 sin (1800π t ) cos ( 400π t ) 1 1 1 = 2 sin (1800π t ) + 4 s i n (1400π t ) + 4 si n ( 2200π t ) Now Rx (τ ) = Rx(τ ) + Rx (τ ) + Rx (τ ) where 1 Rx1 (τ ) = 8 cos (1800πτ ) 1 Rx2 (τ ) = 32 cos (1400πτ ) 1 Rx3 (τ ) = 32 cos ( 2200πτ ) Therefore, 1 1 1 Rx (τ ) = 8 cos (1800πτ ) + 32 cos (1400πτ ) + 32 cos ( 2200πτ ) ⎧ 1 1 1 ⎫ Gx ( f ) = ℑ { Rx (τ )} = ℑ ⎨ 8 cos (1800πτ ) + 3 2 cos (1400πτ ) + 3 2 cos ( 2200πτ ) ⎬ 1 1 = 16 [δ ( f − 900) + δ ( f + 900) ] + 64 [δ ( f − 700) + δ ( f + 700) ] 1 + 64 [δ ( f − 1100) + δ ( f + 1100) ] The normalized average power is obtained by using (2.172) as ∞ 1 ∞ 1 ∞ P = G ( f )df = 16 ∫ [δ ( f − 900) + δ ( f + 900) ]df + 64 ∫ [δ ( f − 700) + δ ( f + 700) ] df −∞ −∞ −∞ 1 ∞ 1 1 1 3 + 64 ∫ [δ ( f − 1100) + δ ( f + 1100) ] df −∞ = 8 + 32 + 32 = 16
  • 90. 46
  • 91. PowerSpectralDensi ty 0. 1 0. 09 0. 08 0. 07 0. 06 0. 05 0. 04 0. 03 0. 02 0. 01 0 -1 -0. 8 -0. 6 -0. 4 -0. 2 0 0. 2 0. 4 0. 6 0. 8 1
  • 92. Contemporary Communication Systems 1st Edition Mesiya Solutions Manual Download:http://testbanklive.com/download/contemporary-communication-systems- 1st-edition-mesiya-solutions-manual/ contemporary communication systems mesiya pdf download contemporary communication systems mesiya download contemporary communication systems pdf contemporary communication systems mesiya solutions 47